Bridging Course for AMC

Share Embed Donate


Short Description

Samples of questions perfect for preparation of AMC MCQ Exam...

Description

Bridging Course MCQs

Block 1 Block 2 Block 3 Block 4 Block 5 Block 6 Block 7 Block 8 Block 9 Block 10

Block 1 Question 3

Question 1 Patients suffering from schizophrenia may experience both positive and negative symptoms. All of the following are negative symptoms EXCEPT: a) Anhedonia b) Hallucinations c) Lack of motivation

Maud is a 70 year old who presents with acute periumbilical abdominal pain gradually increasing in intensity. She is vomiting profusely and develops watery diarrhoea with flecks of blood after an hour of pain. Examination of the abdomen reveals localised periumbilical tenderness with some rigidity. Rectal examination is normal. An irregular pulse is noted and an ECG is recorded (shown below). The MOST LIKELY diagnosis is:

d) Emotional blunting

a) Acute appendicitis

e) Alogia

b) Acute pancreatis c) Perforated peptic ulcer

Answer: B Hallucinations are abnormal perceptions, e.g. hearing voices speaking about the patient, and/or giving instructions to do certain things. They represent additional phenomena and, therefore, are classified as positive symptoms. Delusions (false ideas) is the other major group of positive symptoms. In contrast, each of the other options represents a loss or diminution of a mental function, e.g. anhedonia is a loss of capacity for pleasure.

d) Biliary colic e) Mesenteric artery occlusion

Answer: E The clinical presentation is typical of mesenteric artery occlusion. This occurs most commonly in patients with atrial fibrillation leading to embolism. The ECG shows atrial fibrillation. Arteriography will show the vascular occlusion.

Question 2 Martha was born at 35 weeks gestation. She required phototherapy for jaundice whilst in hospital. Martha is being breast-fed and is beginning to gain weight. She is now two months old. Regarding routine childhood vaccination which of the following statements is CORRECT? a) Vaccinations due to be given at two months of age should be postponed until Martha is at least 13 weeks old to allow for her prematurity. b) Martha's past history of jaundice is not a contraindication to oral polio vaccine. c) Vaccines for intramuscular injection should be given into the buttock in a premature baby. d) Breast-feeding vaccination.

is

a

contraindication

to

e) It is not necessary to immunise Martha against hepatitis B.

Answer: B A history of jaundice after birth is not a contraindication to any of the vaccines in the standard schedule. Vaccination should not be postponed because of prematurity. The fact that a child is breast-fed is not a contraindication to vaccination. Vaccines should never be given into the buttocks. The anterolateral thigh is the preferred site for vaccination in infants under 12 months of age. The deltoid region is the preferred site in older children (those who have commenced walking) and in adults. Martha would be due to receive vaccination against hepatitis B at the age of two months according to the current standard vaccination schedule.

Question 4 A 15 year old male has sudden onset of severe pain in his right lower abdomen commencing 2 hours ago. He has vomited several times in the last hour. He is rolling on the bed, stating that the pain is going down into his groin. T 37.1 degrees Celcius, P 110min, BP 135/ 80. Abdomen - soft, no rebound. Tender right testicle. Your immediate management is: a) i/v fluids and antibiotics b) arrange urgent ultrasound examination c) i/v metoclopramide (maxolon) d) refer for emergency surgery e) arrange for intravenous pyelogram (IVP)

Answer: D The sudden onset of severe pain in the lower abdomen, groin or scrotum, in a young male under 25 years, should be considered to be testicular torsion until proved otherwise. This is a surgical emergency, as infarction of the testis can occur quickly, and surgical exploration should be undertaken urgently. This patient has no fever, nor tenderness of the epididymis to indicate epididymoorchitis. Antibiotic treatment will not help. Colour doppler ultrasound may show increased blood flow in infection and the absence of flow in advanced torsion. However, these are not reliable findings, and the investigation would waste valuable time. The vomiting is related to the pain, and would be alleviated by appropriate analgesia. Metoclopramide is not an

immediate priority. The clinical picture is highly suggestive of testicular torsion rather than renal colic, thus IVP is not the appropriate immediate management.

c) Descent is unlikely to occur after 1 year of age.

Question 5

e) The undescended testis is at reduced risk of malignancy.

In a 3 year old child with signs and symptoms suggestive of bacterial meningitis, which of the following is the BEST initial management? a) Erythromycin IV

d) Orchidopexy should be delayed until late childhood.

Answer: C Testes which are undescended at birth may well descend into the scrotum during the first two weeks of life, however descent is unlikely to take place after the age of one

b) Gentamicin IV c) Ceftriaxone IV d) Phenoxymethylpenicillin oral e) Amoxycillin oral

Answer: C If bacterial meningitis is suspected clinically it is vital to immediately administer an appropriate antibiotic prior to urgent transfer to hospital, as meningococcal meningitis may be rapidly fatal. The drug of choice would be benzylpenicillin 60mg/kg up to 3g IV or IM, or ceftriaxone 50mg/kg up to 2g IV in patients hypersensitive to penicillin or when further drug treatment may be delayed.

year. 2% of boys born at full-term, and 20% of premature males, have undescended testes. A testis which was palpable in the scrotum in infancy may ascend and become impalpable due to failure of the spermatic cord to elongate at the same rate as body growth. Orchidopexy is best performed by 12-18 months of age as spermatogenesis in the undescended testis is impaired after the age of two years. The undescended testis is at 5-10 times greater risk of developing malignancy (seminoma).

Question 8 An aspirate in an acutely painful, swollen knee shows the following:

Question 6

• white cell count 4100/uL (< 2000/uL)

Atypical antipsychotic drugs (eg. olanzapine) have certain advantages compared to the typical antipsychotic drugs (eg. chlorpromazine) in the treatment of schizophrenia. All of the following are advantages of atypical drugs EXCEPT:

• red blood cells ++

a) Improved therapeutic effect on positive symptoms b) Improved therapeutic effect on negative symptoms c) Reduced potential for acute extrapyramidal symptoms d) Reduced potential extrapyramidal symptoms

for

e) Improved therapeutic treatment-resistant patients

effect

longer-term in

some

Answer: A Both the typical and atypical antipsychotic drugs have a beneficial effect on positive symptoms in schizophrenia. However, they differ with regard to negative symptoms. Atypical antipsychotics are much better at combating these. The other options are true for the atypical drugs.

• no organisms cultured What is your interpretation of these results? a) Inflammation is more likely b) Traumatic tap - cannot be interpreted c) Gonococcal arthritis d) Tuberculosis arthritis e) Viral arthritis

Answer: A Normal synovial fluid contains less than 2000 white blood cells per microlitre. Inflammation causes counts of 3000 or higher. Inflammation can also cause red blood cells to migrate into the joint fluid.

Question 9 Concerning prostate cancer, which ONE of the following is INCORRECT? a) The majority of carcinomas arise in the peripheral zone of the gland

Question 7 Which of the following statements undescended testes is CORRECT?

• no crystals

regarding

a) The testes are undescended at birth in 40% of boys. b) Once the testis is palpable in the scrotum it will remain so.

b) The prostate specific antigen (PSA) is elevated in all prostate cancers greater than 1.5 cm c) Metastatic spread to pelvic lymph nodes occurs early

d) Radionuclide bone scan is not a reliable method of detecting bony metastases e) Prostate cancers usually appear as hypoechoic nodules on transrectal ultrasound

Answer: B A normal PSA occurs in up to 20% of cancers, including tumours greater than 1.5 cm. Prostate cancer is the second most common cause of cancer deaths in men in Australia. The majority of prostate cancers- usually adenocarcinomas- arise in the peripheral zone of the gland, and metastasise early to pelvic lymph nodes. Bony metastases are also common. Bone scan is unreliable at detecting bony metastases when the prostate specific antigen (PSA) is 150mg/kg/day were necessary for hepatotoxicity but sick children 90mg/kg/day for more than one day are at higher risk

Question 26 All of the following amenorrhoea EXCEPT:

Answer: E are

causes

of

secondary

a) Diabetes

Studies indicate that maternal transmission of HIV to the infant is most likely in the perinatal period. The other options are correct. Advanced maternal disease is also a risk factor for increased transmission risk.

b) Drug abuse c) Eating disorders

Question 29

d) Asherman's syndrome

Which of the following statements concerning a retained placenta is CORRECT?

e) Autoimmune disorders

a) antibiotic cover is rarely necessary after a manual removal of placenta Answer: A Secondary causes of amenorrhoea and oligomenorrhoea include eating disorders, drug abuse, Asherman's syndrome (intrauterine adhesions), thyroid or adrenal dysfunction, uterine or vaginal obstruction, familial early menopause, chromosomal abnormalities and autoimmune disorders. Diabetes is not a cause.

Question 27 Muriel, aged 80 years, fell onto her outstretched right hand sustaining a Colles' fracture. Which of the following is the most frequent LATE complication of this fracture for a patient of her age? a) delayed union of the fracture b) ischaemic necrosis of the distal fragments c) neuralgic pain in the arm and hand d) stiffness of the wrist and fingers e) ulnar nerve weakness

Answer: D Joint stiffness is common following a Colles' fracture, especially after prolonged immobilisation. All of the other complications are relatively uncommon. Union is usually not delayed but there may be a degree of malunion. Appropriate management of the elderly patient with a Colles' fracture includes early mobilisation aimed at restoring function.

Question 28 Amanda, aged 24 years, is HIV positive. She has just given birth to twin sons, George and Harry, by normal vaginal delivery. Regarding vertical transmission of HIV from mother to baby, which ONE of the following statements is INCORRECT? a) the first born twin is more likely to be infected than is the second born twin b) higher rates of transmission are likely with low maternal CD4 lymphocyte count c) breast feeding is a possible route of vertical transmission d) high maternal plasma HIV RNA increases transmission risk e) transmission to the foetus is most likely in the antenatal period

b) the third stage of labour takes on average two hours to complete c) when delivering the placenta the doctor or midwife should pull the cord upwards d) there is no association between placenta praevia and placenta accreta e) oxytocics and continuous cord traction enable delivery of most placentas within 10 minutes

Answer: E The third stage of labour (when the placenta is delivered) is normally completed within 30 minutes. If not, it is unlikely to occur spontaneously. With the use of oxytocics and gentle continuous downward cord traction, 97% of third stages are complete within 10 minutes. Antibiotic cover is necessary for manual removal of the placenta, because of passage of the hand from a non-sterile (vagina) to a sterile (uterus) environment. Placenta accreta (where placental villi penetrate the uterine wall preventing separation) occurs more commonly in cases of placenta praevia and after caesarean section.

Question 30 Hazel, aged 40 years, has a month-long history of a burning pain in the middle of her right foot. She says it's "as though I'm standing on a sharp stone". The pain radiates into the tips of her toes. Pressure applied to the dorsum of the head of the third metatarsal reproduces her pain, but there is no other abnormality noted. What is the MOST likely diagnosis? a) Interdigital (Morton's) neuroma b) Plantar fasciitis c) Gout d) Peripheral neuropathy e) Stress fracture of the third metatarsal

Answer: A Interdigital neuroma represents a gradual and persistent thickening of the perineurium of one, or less commonly two or more interdigital nerves. Hazel's history is typical, as is the examination finding of reproduction of the pain by direct pressure over over the head of the metatarsal. Plantar fasciitis causes primarily heel pain.

The absence of heat, redness or swelling over the metatarsal heads makes gout and rheumatoid arthritis unlikely. Peripheral neuropathy is usually associated with nocturnal paraesthesia, dysaesthesia and burning pain across the sole of the foot but is unlikely to be affected by dorsal pressure on a metatarsal head. Stress fractures of the metatarsals cause pain that is felt along the shafts of the metatarsals, and is unlikely to radiate to the tips of the toes.

Question 31

C. The problem is not an exacerbation of the symptoms of a chronic condition (e.g., major depressive disorder). D. The above criteria must be confirmed by prospective daily ratings during at least three consecutive symptomatic cycles to confirm a provisional diagnosis.

Question 32

With regard to symptoms of the severe form of the premenstrual syndrome (premenstrual dysphoric disorder) which of the following is CORRECT? The symptoms: a) are confined to the late luteal phase of the menstrual cycle b) may eventually extend throughout the whole menstrual cycle c) disappear within hours of the onset of menstruation d) may fluctuate and disappear altogether in some cycles e) may present as endogenous depression

B. Symptoms cause marked interference with work, school, usual social activities, or relationships with others.

an

exacerbation

of

Classic symptoms of endometriosis include all of the following EXCEPT: a) pelvic pain b) dysmenorrhoea c) dyspareunia d) oligomenorrhoea e) infertility

Answer: D Endometriosis is the presence of endometrial tissue outside the uterine cavity. It causes pelvic pain, dysmenorrhoea, dyspareunia and infertility. Oligomenorrhoea is not associated with endometriosis.

Answer: A While premenstrual syndrome is the term commonly used to describe the constellation of symptoms prior to periods, Premenstrual Dysphoric Disorder is a much more specific condition listed by the American Psychiatric Association in the Diagnostic and Statistical Manual of Mental Disorders (DSM-IV). The diagnostic criteria for PMDD are: A. At least five of the following symptoms (one of which must be 1, 2, 3 or 4, below) must be present in the majority of menstrual cycles in the last year. Symptoms should be isolated to the late luteal phase of the menstrual cycle and remit within days of onset of menses. 1. Markedly depressed mood, feelings hopelessness, self-deprecating thoughts

of

2. Marked anxiety, tension, feelings of being "keyed up" or "on edge" 3. Marked affective lability 4. Persistent and marked anger or irritability or increased interpersonal conflicts 5. Decreased interest in usual activities 6. Subjective sense of difficulty concentrating 7. Lethargy, easy fatigability, marked lack of energy 8. Marked change in appetite 9. Marked change in sleep pattern 10. Subjective sense of being overwhelmed or out of control 11. Physical symptoms (e.g., breast tenderness or swelling, headaches, joint or muscle pain, sensation of bloating, weight gain)

Question 33 For a perimenopausal woman who has irregular cycles, severe hot flushes and no contraindications to hormone replacement therapy the MOST APPROPRIATE therapy is: a) continuous oestrogen therapy b) continuous combined progestogen) therapy

(oestrogen

and

c) sequential oestrogen therapy d) sequential combined progestogen) therapy

(oestrogen

and

e) continuous progestogen therapy Answer: D Hormone replacement therapy (HRT) is indicated in women who are suffering from severe menopausal symptoms, provided they have no contraindications to its use. Sequential combined HRT is the best option for perimenopausal women who do not require contraception and for women in the first 2 years after menopause. It can alleviate symptoms and control irregular cycles. The use of continuous or sequential unopposed oestrogen is associated with endometrial hyperplasia and the development of endometrial cancer and is contraindicated in women who have not had a hysterectomy. Continuous combined HRT is recommended for symptomatic women more than 2 years post menopause; 50% will have irregular bleeding for the first 6 months but 90% are amenorrhoeic after 12 months. Progestogen alone is not always effective at treating menopausal symptoms. The issues surrounding potential complications of HRT must always be fully discussed before commencing therapy.

Question 34

Question 36

Sherri, aged 56 years, complains that she loses control of her "waterworks" when she puts her key in the front door. Given this history, which form of incontinence is she MOST LIKELY to have?

Cheryl, aged 28 years, (G1P0), presents at 8 weeks gestation complaining of constant nausea and vomiting. Which of the following statements about nausea and vomiting in pregnancy is CORRECT?

a) stress incontinence b) urge incontinence c) overflow incontinence d) incontinence from a urinary fistula e) incontinence secondary to a neuropathic bladder

Answer: B Urge incontinence occurs when there is an inability to delay micturition and may be precipitated by various triggers including the sound of running water or placing a key in the door when arriving home. Stress incontinence occurs when the intra-abdominal pressure is raised as with coughing or sneezing. Overflow incontinence is due to obstruction and may be secondary to uterovaginal prolapse or a hypotonic bladder as in a neuropathic bladder. A urinary fistula is associated with continuous dribble of urine or leakage of small amounts on effort.

Question 35 Teresa, aged 25 years, presents having experienced an episode of postcoital bleeding two days ago. What is the MOST APPROPRIATE management? a) Reassure her and ask her to return if bleeding recurs b) Undertake cauterisation of the cervix to prevent further bleeding c) Treat her with metronidazole gel to eradicate infection d) Send her to the emergency department for immediate assessment e) Do a Pap smear and screen for sexually transmitted infections

a) Less than 30% of women suffer nausea and vomiting in pregnancy b) Metoclopramide (antiemetic) contraindicated in pregnancy

is

c) Women should try to have frequent small feeds to control nausea d) Nausea and vomiting should subside by 9 weeks gestation e) Nausea and vomiting in the evening points to a more sinister cause

Answer: C At least two thirds of women experience nausea during the first trimester, and 50% experience vomiting. Symptoms can occur at any time of the day, although classically they predominate in the morning. These symptoms usually subside by 12-16 weeks gestation. The best advice for women is to take small frequent meals but, if the symptoms persist antiemetics such as metoclopramide can be used safely.

Question 37 Which of the following elements on an antenatal cardiotocograph (CTG) at term is considered "abnormal"? a) Accelerations of 15 beats per minute lasting 15 seconds b) One reactive movement in a 20 minute CTG recording c) Decelerations contractions

during

Braxton

Hicks

d) Variability in foetal heart rate beat-to-beat e) Baseline heart rate of 120-160 beats per minute

Answer: E

Answer: C

Postcoital bleeding is a serious symptom that could be indicative of cervical pathology. It is not an emergency requiring assessment in hospital. Common causes of postcoital bleeding include a cervical erosion, an infection such as chlamydia and other less common pathologies, such as a cervical polyp. Medical practitioners must however ensure that they exclude precancerous or cancerous lesions of the cervix by making sure that cervical cytology (Pap smear) is performed as well as appropriate STI (sexually transmitted infection) screening. If the bleeding is recurrent, or the cervix looks abnormal, colposcopy is recommended. Cauterisation of the cervix is sometimes performed if a friable cervical erosion is present, bleeding is recurrent and other cervical pathology has been excluded.

Decelerations occurring after contractions are ominous, particularly if they are prolonged. A healthy CTG shows a baby that is moving and having acceleration of heartbeat after movement. The beat to beat variability is indicative of an intact central nervous system.

Question 38 Melinda has just started to menstruate (experience periods). She is age 16 and has Down syndrome (Trisomy 21). She wants to know all about her periods and why she has to bleed. Which of the following statements should you NOT tell Melinda regarding menstruation (periods)?

a) Most females start having their periods between the ages of 9 and 16 b) Other body changes may be happening when periods start. Breasts get bigger and hair starts to grow under arms and around the vagina c) A period will occur about once a month and will last for several days (about 3 to 7 days) d) The blood that comes out with a period is clean and healthy and it is normal e) Tampons must be used during periods to avoid bleeding onto underwear and clothes

malpresentations, polyhydramnios, during breech deliveries and with premature rupture of the membranes It is an obstetric emergency, as the umbilical vessels constrict, once exposed to the extrauterine environment. Unless the cervix is fully dilated and an immediate operative vaginal delivery can be conducted, an emergency caesarean section is required. During the transfer to theatre the woman should be positioned so that gravity can assist in keeping the presenting part off the cord, i.e the knee - chest position (kneeling with head down). The presenting part should also be pushed digitally up and away from the cord which should be placed wholly within the vagina.

Question 40

Answer: E All the other statements are true and helpful when a female with a potential learning disability starts learning about her menstruation. Tampons can be particularly convenient for certain activities, such as swimming, but they are not essential and are not recommended for women who cannot comfortably manage their own menstrual self-care. If tampons are to be used it is worth checking instructions are understood before encouraging practice.

In counselling a woman regarding use of the mini pill (progestogen only contraceptive pill, POP) which of the following is CORRECT? a) The menstrual cycle will be unaffected by the POP b) The main mode of action is to thicken cervical mucous c) Active pills are taken for 21 of 28 days to allow a withdrawal bleed

Question 39

d) A delay of 12 hours in taking the POP does not affect efficacy

Angelina, aged 27 years (G3 P2), has a transverse lie at 36 weeks gestation. Should her membranes rupture, what would be the MOST APPROPRIATE MANAGEMENT?

e) Contraceptive efficacy of the POP equivalent to combined pill in all women

a) Advise attendance at the Delivery Suite when contractions are 5 minutes apart b) Attempt an external cephalic version to allow safer vaginal delivery c) Wait for contractions to establish as this will probably correct the lie d) Augment the labour with oxytocin to facilitate swift delivery

intravenous

e) Place in the knee-chest position and organise emergency Caesarian section

Answer: E Cord prolapse occurs when the umbilical cord lies beside or in front of the presenting part. It is more common in

is

Answer: B The principal mode of action of the progestogen only pill is thickening of cervical mucus. In about one third of women the minipill will also inhibit ovulation but in the majority this is not the case. As a result of the variability of the effect of the POP on ovulation and the effects of progesterone on the endometrium, menstrual cycles may be regular, irregular or spotting can occur throughout the cycle in POP users. Women take the minipill everyday without a break (28 active pills with no inactive pills) in the same three-hour period each day in order to maintain maximal efficacy. In general, it is less efficacious than combined oral contraception because it does not uniformly inhibit ovulation. However, in older women who are less fertile and who use the POP correctly, the efficacy of the POP can approximate that of combined oral contraception.

Block 3 Question 1

Question 3

Tom, aged 18 years, presents with a typical first episode of schizophrenia. Which of the following antipsychotic medications is the treatment of choice for him?

Sarah, a 40 year old woman whose husband has a plasma cholesterol of 6.9 mmol/l (normal 85 suffers from dementia b) Dementia affects one in four people aged 80-85

Question 39 Malcolm, aged 25 years, presents complaining of feeling unwell with a painful ulcer on his penis. He has tender inguinal lymphadenopathy on examination. What is the MOST LIKELY diagnosis? a) Primary genital herpes

c) Family history is a major risk factor for Alzheimer disease

b) Primary syphilis

d) Vascular disease is the most common cause of dementia

d) Recurrent genital herpes

e) Dementia is no more common in the indigenous population than in the general community

Answer: C Age and family history are the two most common risk factors for dementia. One in 15 Australians aged 65 and over has dementia. In people aged 80-85 years, it affects 1 in 9 people. In those over 85 years, it affects 1 in 4. Alzheimer's disease is the most common cause of dementia. In the most recent assessment of indigenous Australians, 10% of those aged 65 and over were found to have dementia and another 10% were suspected of having it.

c) Secondary syphilis

e) Chancroid

Answer: A Primary genital herpes is the most likely cause of a painful ulcerative lesion on his penis. It begins as multiple vesicles which ulcerate and can become secondarily infected. Recurrent genital herpes episodes tend to become milder and less frequent over time. The primary lesion of a syphilitic ulcer is painless and usually persists for 4-6 weeks and heals spontaneously. Chancroid produces multiple painful exudative nonindurated ulcers.

Question 40

Answer: C

A 24 year old married woman presents with patches of a scaly coppery pink macular rash over the trunk. The rash has been present for one week. The patches are oval, of different sizes, and appear to be spreading. The patches are arranged along the skin creases. She feels well. There are no other abnormal findings. Which of the following is the MOST APPROPRIATE management?

The description is typical of Pityriasis rosea and management is usually reassurance only as this is a self-limiting condition and disappears in 4-10 weeks. Calamine lotion can be used if there is an associated itch and topical steroids are only rarely used in the presence of moderately severe itch.

a) Application of benzyl benzoate lotion b) Prescription of antihistamines c) Reassurance as it is a self limiting condition d) Pathology test for rubella antibody titre e) Application of topical steroids

Block 4 Question 1 Which of the following statements about Dupuytren's contracture is CORRECT? a) It is a thickening of the tendon sheath b) It usually causes flexion contracture of the 2nd and 3rd fingers of the hand c) It is more common in women than men d) The mode of inheritance recessive

is autosomal

e) It is more common in diabetics

breathe, heavy in the chest, clammy and her heart races. These symptoms also occur at other times quite unexpectedly. These 'turns' last about 10 mins and then she feels better, but Mary is really scared about when they might occur next. What is the MOST LIKELY diagnosis? a) Mary has anxiety b) Mary has a phobia c) Mary has anxiety with panic attacks d) Mary is depressed and has a phobia for shopping centres e) Mary has anxiety, and ongoing angina

Answer: E Dupuytren's contracture is a thickening of the palmar fascia resulting in flexion contracture of the fingers of the hand particularly the ring and fifth fingers. Its aetiology is unknown, but it is thought to be familial. It is more common in men than women. It is more common in alcoholics, diabetics and epileptics treated with phenytoin.

Answer: C Mary has the classical symptoms of panic attacks with some pre-existing anxiety. With panic attacks symptoms must peak within 10 min and usually dissipate within minutes, leaving little to observe, except the person's fear of another terrifying panic attack. A distinguishing feature of panic disorder is that some of the panic attacks are unexpected or spontaneous.

Question 2 Which of the following drugs is contraindicated in a child with known glucose-6-phosphate dehydrogenase (G6PD) deficiency? a) Paracetamol b) Salbutamol c) Metronidazole d) Sulphamethoxazole e) Prednisolone

Question 4 Clarice, 26 years, presents to you concerned because she has noticed that a dark mole on her thigh has become enlarged, slightly lumpy and itchy over the last two months. The MOST APPROPRIATE initial management would be to: a) Ask Clarice to return for review in three months b) Take an incisional biopsy of the lesion for histopathology c) Treat the lesion using liquid nitrogen

Answer: D

d) Remove the lesion using laser

Glucose-6-phosphate dehydrogenase deficiency is a genetically inherited enzyme deficiency which results in acute haemolysis upon exposure to an environmental stress (viral or bacterial infections), drugs or toxins. Particular drugs may cause haemolysis in patients deficient in G6PD including Sulphamethoxazole, nitrofurantoin, primaquine, aspirin and others. Fava beans may also precipitate haemolysis in a minority of patients.

e) Undertake an elliptical excision clear of the margin for histopathology

Question 3 It has been 18 months since Mary had her heart attack and stroke. She is 81 years old, slightly anxious, but very independent & mobile, even though she gets a little short of breath going up stairs. Increasingly, Mary is fearful of leaving the house, because sometimes as she is due to leave, she feels dizzy, unsteady, unable to

Answer: E If a malignant melanoma is suspected then an accurate pathological report is required to guide further management. For this reason it is important that the initial management involves complete removal of the lesion without destruction of the tissue. Early detection and removal of melanomas leads to better outcomes (Clark's level one and two melanomas have a five year prognosis of >90%). If a melanoma is diagnosed then referral to a plastic surgeon is necessary for a wide local excision involving a margin of 1-3 cm and to a depth of the deep fascia.

Question 5 All of the following may be features of Down syndrome (Trisomy 21) EXCEPT:

deviation. As the apparent right ventricular hypertrophy disappears the ECG takes on a more adult appearance, and should have an adult pattern by the age of ten years.

a) Hypotonia b) Webbing of the neck

Question 8

c) Congenital heart defects

Benny has always loved to go clubbing, and often after a few drinks at the end of a night of dancing, he ends up having casual sex with someone he meets at the nightclub.

d) Abnormalities of the dermal ridge pattern e) Epicanthic folds

Answer: B Webbing of the neck is a feature seen in patients with Turner syndrome. All other listed features may be present in a patient with Down syndrome.

Benny had his first hepatitis B serology testing done last week. These are his test results: • HBsAg = positive • HBsAb = negative • IgM HBcAb = positive • HBeAg = positive.

Question 6 The clinical features associated with raised intracranial pressure include all of the following EXCEPT: a) morning headache b) vomiting c) presence of papilloedema d) decrease in conscious state e) falling blood pressure with a falling pulse

What is the MOST LIKELY cause of these results? a) Benny has been vaccinated in the past for hepatitis B and is now immune b) Benny has had hepatitis B infection sometime in the past and it has resolved, leaving him with life-long immunity c) Benny is a hepatitis B carrier d) Benny has acute or current hepatitis B infection e) Benny has early liver cirrhosis

Answer: E Rising blood pressure (not falling) in combination with a falling pulse rate is a classical feature of rising intracranial pressure known as the Cushing response. Headache occurs as a result of the deformation of intracranial blood vessels and dural membranes which arises from conditions which give rise to raised intracranial pressure. The headache is worst in the morning (as is vomiting) and is aggravated by coughing, sneezing or stooping. When present papilloedema (swelling of the nerve fibres of the optic disc) is highly suggestive of raised intracranial pressure. A decrease in conscious state commencing with confusion and progressing through various grades of coma is also seen with increasing intracranial pressure.

Answer: D Benny is HBsAg positive which occurs 1-6 months after exposure to the hepatitis B virus and indicates acute infection. If HBsAg persists after 6 months, it defines carrirer status. HBsAb is not present (it would be positive following vaccination). IgM HBcAb is present in acute infection only (IgG HBcAb is present in highly infective carriers and in acute infection). HBeAg is present and implies high infectivity in recent infection and carriers. Benny needs education about hepatitis B, safe sex & drug use.

Question 9 Question 7 In the first year of life which of the following ECG features may be considered normal? a) Right axis deviation b) Sinus bradycardia c) First degree heart block d) Left bundle branch block e) ST segment depression

Answer: A At birth the right ventricular muscle is as thick as the left. This results in an ECG pattern which would indicate right ventricular hypertrophy, including right axis

Which of the following is FALSE regarding neural tube defects and folate before and during pregnancy? a) Folate intake should be increased at least one month before and three months after conception b) Most women before and during pregnancy need 0.5mg folate daily c) Women on anti-epileptic medication may require 5mg folate daily before and during pregnancy d) Folate reduces the incidence of neural tube defects which occur at the rate or 1:5000 pregnancies e) Women with a family history of neural tube defects need more folate before and during pregnancy

Answer: D

Question 12

Pregnant women are at increased risk of folate deficiency due to the high demand of the developing foetus. Deficiency in the first few weeks of pregnancy can cause neural tube defects in the newborns. Neural tube defects occur at a rate of 1:500 pregnancies. The other options are true.

Kylie is pregnant. She has smoked 25 cigarettes per day for the past 10 years and continues to smoke even now she is pregnant. Early in her pregnancy you outline to her, that, compared with infants born to non-smoking mothers, her infant is more likely to experience a number of disadvantages. These include all of the following EXCEPT: a) Higher perinatal mortality

Question 10 Pamela aged 45 years, attends having found a lump in the upper outer quadrant of her right breast two days ago. She is concerned about the likelihood of cancer. In order to diagnose the nature of the lump you invoke the use of the "triple test" or "triple assessment". The triple test consists of: a) Clinical examination, magnetic resonance imaging b) Mammography, biopsy

mammography,

ultrasound,

fine

needle

c) Clinical examination, mammography, fine needle biopsy d) Clinical examination, ultrasound, magnetic resonance imaging e) Ultrasound, fine needle biopsy, magnetic resonance imaging

Answer: C Management of breast lumps is now based on the triple test, which combines the results of clinical examination, mammography (+/- ultrasound) and fine needle aspiration biopsy. When combined, these tests give a sensitivity of 95-99% in the diagnosis of breast lumps.

b) Small for gestational age c) Greater likelihood of sudden infant death syndrome d) Developmental lag at least in early years e) Greater likelihood of developing small teeth with faulty enamel

Answer: E Small teeth with faulty enamel is a disorder resulting from excess alcohol intake in pregnancy and the foetal alcohol syndrome. The other options are all disadvantages experienced by infants of women who smoke.

Question 13 Alison, aged 18 years presents with a mobile, smooth, solid lump of 2 cm diameter in her left breast. The MOST LIKELY diagnosis is: a) malignancy b) fibroadenoma c) breast cyst d) intraductal carcinoma

Question 11 On examining Fatima (aged 18 months), whom you are seeing for the first time, you hear a heart murmur. Which of the following clinical findings would suggest that this is an innocent heart murmur? a) The murmur is diastolic b) The murmur is associated with a thrill c) The murmur is pansystolic d) The murmur disappears when the child lies down e) The murmur is associated with reduced exercise tolerance

e) breast abscess

Answer: B A fibroadenoma is a benign breast condition that arises as an aberration of normal development and involution. The diagnosis is best made using the triple test. Indications for removal include patient preference or discomfort, size > 3 cm, continued growth or lump presenting for the first time > 40 years. Fibroadenomas may spontaneously disappear or calcify.

Question 14 Which of the following statements concerning nappy rash is CORRECT?

Answer: D The disappearance of the murmur when the child lies down suggests it is the innocent murmur known as 'venous hum'. This is a murmur produced by blood flow through the great veins and is heard at the base of the heart, often just below the clavicles. It is blowing and continuous in nature. The murmur varies with respiration and the position of the head, and disappears when the child lies down. The other features listed are not those of an innocent murmur.

a) Seborrhoeic dermatitis is the most common cause b) Management should include liberal use of talcum powder to help keep the nappy area dry c) The napkin area should be washed frequently with soap to avoid recurrences d) Superinfection with Candida tends to involve the flexures e) Topical treatment

steroids

are

the

mainstay

of

Answer: D

a) The usual age of symptom onset is 2-6 years

Irritant dermatitis is the most common cause of nappy rash and tends to spare the flexures. Candidiasis will involve the flexures and may extend beyond the napkin area as 'satellite lesions'. In managing nappy rash, the area should be kept dry, but powders should be avoided as should soaps and excessive bathing or scrubbing. Topical corticosteroids should be used with caution to treat specific causes of nappy rash only, including atopic dermatitis and seborrhoeic dermatitis.

b) Genetic recessive

inheritance

is

usually

X-linked

c) Approximately 25% of patients die by the age of 20 years d) A lordotic, waddling gait is a feature e) Female carriers are usually asymptomatic

Answer: C

Question 15 The classical signs of congenital rubella measles) include all of the following EXCEPT:

(German

75% of patients with Duchenne muscular dystrophy die by age 20, usually from cardiac or respiratory failure. The other options given are correct for this condition.

a) Cataract b) Heart disease

Question 18

c) Deafness

Mel always loved playing girls games as a child and had fantasies about being female. When puberty arrived he was distressed at the physical changes that occurred in his body, and as soon as he left home he adopted a complete female appearance and female role in public and private. He obtained a driver's license and was able to work and live in society as a woman. Mel takes ethinyl estradiol 0.10 mg/day and has nearly completed 2-years of living completely as a woman. Mel has requested sex reassignment surgery. What is Mel's diagnosis?

d) Low birth weight e) Koplik's spots

Answer: E Koplik's spots are typically associated with measles (rubeola) only and not any other infectious diseases. The other options are features of congenital rubella.

a) Transvestite b) Cross-dressing homosexual

Question 16 Which of the following statements regarding carcinoma of the lung is CORRECT? a) Lung cancer is the most common registrable cancer in women b) Approximately 10% of lung cancers are derived from squamous cells c) Approximately 60% of lung cancers are adenocarcinomas d) The majority of lung asymptomatic at diagnosis

cancers

are

e) Exposure to asbestos increases risk of lung cancer

Answer: E Smoking and exposure to asbestos are associated with the development of squamous cell and adenocarcinoma of the lung. Prostate and breast cancer are the most common registrable cancers in men and women respectively. Lung cancer is the most common malignancy causing death in men and second most common in women after breast cancer although incidence in women is rising. Of lung cancers, 32% are adenocarcinoma, 29% are squamous cell, 9% undifferentiated large cell and 18% small cell type. Up to 15% of people are asymptomatic of their lung cancer at diagnosis.

Question 17 Which of the following statements regarding Duchenne muscular dystrophy is INCORRECT?

c) Schizophrenia with gender issues d) Borderline personality disorder e) Male transsexual

Answer: E Male transsexualism is a gender identity disorder in which the male believes he is the victim of a biologic accident, cruelly imprisoned in a body incompatible with his subjective gender identity. Transvestism occurs when heterosexual males dress in women's clothing, and at least initially this is associated with sexual arousal. Transvestism is a psychiatric disorder only if the fantasies, urges, or cross-dressing behaviours are associated with clinically significant distress or recognizable dysfunction. Cross-dressing per se is not a disorder. Homosexuality is not a psychosexual disorder but a preference of a sexual partner. Schizophrenia is not a gender disorder.

Question 19 Regarding Sudden Infant Death Syndrome (SIDS), which of the following statements is CORRECT? a) Positioning a baby prone (on its front) to sleep may reduce the risk b) The incidence is greater in female infants c) Maternal smoking has not been shown to be a risk factor d) Breastfed infants are at greater risk e) Infant overheating may be a risk factor

Answer: E Regarding modifiable risk factors for SIDS, positioning the infant supine (on its back) to sleep, breastfeeding, avoidance of overheating, and maternal smoking cessation may reduce risk. Male infants are more at risk from SIDS (the male: female ratio is approximately 3:2).

trauma or spontaneously. The physical signs of a pneumothorax are a hyper-resonant percussion note and absent breath sounds. In cases of tension pneumothorax, there is mediastinal displacement away from the side of the defect. Treatment is necessary when there is a large enough pneumothorax to inhibit respiratory activity and involves the insertion of an intercostal drain in the fifth intercostal space in the midaxillary line or alternatively in the second intercostal space anteriorly in the midclavicular line.

Question 20 With regards to cryptorchidism (undescended testes), which of the following statements is CORRECT? a) It is essential that the testes are returned to their normal position in the scrotum by the time the boy is five years old b) Bilateral undescended testes is a more common occurrence than unilateral undescended testis c) The lower the arrest in the line of descent of the testis the more hypoplastic it is d) Malignancy in the undescended testis is 2030 times more common than usual e) Inguinal hernia is associated with undescended testes in approximately 50% of cases

Question 22 Beth, aged 6 months, is brought to see you by her mother who has noticed her eyes are not always lined up. You are concerned Beth may have a squint (strabismus). Which of the following statements regarding strabismus is CORRECT? a) Investigation is unnecessary in this age group as strabismus improves with time b) By the age of 6 months Beth's eyes should be constantly well aligned c) Strabismus is rarely a marker of other ocular disease d) Strabismus is not associated with amblyopia e) The corneal light reflex is a reliable test to diagnose strabismus

Answer: D Complications of undescended testes include defective spermatogenesis, torsion, trauma, and malignant degeneration 20-30 times more common even after surgical placement in the scrotum. Ninety five percent are associated with a patent processus vaginalis but only 25% develop a clinical hernia. Twenty five percent have bilateral undescended testes. The testis/es may be intrabdominal, inguinal or high in the scrotum. The higher the arrest along the line of descent the more hypoplastic the testis. In order to minimise these complications the testis/es should be placed in their normal position before the second year of life.

Question 21 Regarding pneumothorax, statements is CORRECT?

Answer: B A baby's eyes should be constantly well aligned by the age of 5 to 6 months. Intermittent ocular deviation should be investigated if present at six months, as it may be a marker of severe underlying ocular or neurologic disease. It should never be assumed that the strabismus will be outgrown. The corneal light reflex test should not be relied upon to diagnose or exclude strabismus. The cover test is a more accurate diagnostic test. Strabismus may lead to amblyopia, which in turn may result in permanent loss of vision if it is not corrected by 4 to 6 years of age.

Question 23 which

of

the

following

a) It is due to the presence of air outside the parietal pleura b) It may occur spontaneously with or without underlying lung disease c) Clinical examination reveals a dull percussion note and absent breath sounds d) There may be a mediastinal shift towards the side of the pneumothorax e) Treatment consists of an intercostal drain in the second intercostal space in the mid axillary line

Answer: B Pneumothorax is the presence of air between the visceral and parietal pleura. It can occur as result of

Sam and Mary have two daughters. Their second daughter has Cystic Fibrosis, but their elder daughter does not. They are considering having another baby. The likelihood of Sam and Mary having another child with Cystic Fibrosis is? a) 1 in 2 b) 1 in 4 c) 1 in 10 d) 1 in 16 e) 1 in 25

Answer: B Cystic Fibrosis is an autosomal recessive disorder. If a husband and wife are both carriers of the autosomal recessive gene then each pregnancy has a 25% chance of resulting in a child who will be homozygous for and thus affected by the disease.

Question 24 Sean, aged 65 years, presents with a history of painless haematuria over the last week. Possible causes include all of the following EXCEPT: a) Cancer within the kidney b) Use of anticoagulants c) Glomerulonephritis d) Benign prostatic hypertrophy e) Use of cyclophosphamide

Answer: D Benign prostatic hypertrophy is associated with difficulty micturating but not haematuria. Common causes of painless haematuria include malignancy of the renal pelvis, drugs such as anticoagulants, cyclophosphamide and D- penicillamine, and glomerulonephritis. Rarely it can result from a bleeding tendency due to inherited disorders, bleeding secondary to idiopathic thrombocytopenic purpura or Henoch Schonlein disease, malaria, "jogger's haematuria", schistosomiasis.

Question 25 Robyn, aged 43 years, is known to have gallstones. On this occasion she presents with the acute onset of severe pain which was at first central in location but has now moved to the right costal margin and radiates to the back. She is pyrexic, slightly tachycardic and has tenderness over the area of the gall bladder but no rigidity of the abdomen. The MOST APPROPRIATE MANAGEMENT would be to: a) Observe her at home for 2 to 3 days to allow this attack to settle b) Admit her to hospital for treatment with IV fluids and antibiotics c) Avoid opioid analgesia due to the risk of worsening biliary spasm d) Admit her to hospital for urgent surgery e) Treat this episode with the goal of preparing for surgery in 2 to 3 months

Answer: B Robyn has acute cholecystitis. Initial management includes IV fluids and nil by mouth, pain relief with parenteral opiate administration and a short, intensive course of antibiotics. Although opiates may increase biliary spasm this is not a contra-indication in view of their excellent analgesic effect. The patient is monitored and immediate operation is ONLY indicated if the fever does not settle or symptoms worsen, indicating perforation of the gall bladder or peritonitis. Immediate operation is not warranted, as there is no indication of perforation of the gall bladder or peritonitis. However, early operation for acute cholecystitis is now recommended compared to delaying surgery.

Question 26 Kari is 7 months old and has not received any immunisations. She presents with two weeks of paroxysmal coughing and vomiting, but is relatively happy between paroxysms. You suspect she may have whooping cough (pertussis). Kari lives at home with her

mother, father and older brothers, aged 2 and 4 years. Neither of her brothers have been immunised against pertussis. Choose the BEST INITIAL MANAGEMENT option from the list below. a) Arrange to have Kari admitted to hospital and isolated immediately b) Report the family to the child protection agency in your state for failing to immunise their children c) Vaccinate Kari immediately with DTPa-hepB or DTPa d) Prescribe oral erythromycin for Kari and the whole family e) Take a nasopharangeal aspirate for diagnosis, and await confirmation of diagnosis prior to starting any other treatment measures

Answer: D Whilst it is important to obtain a laboratory diagnosis of pertussis, this should not delay treatment, which should be commenced after appropriate nasopharangeal aspirate or serological samples are collected. Kari should be treated with erythromycin 10mg/kg/dose up to 250mg orally 6 hourly for 10 days, as should all household and other close contacts. This will not shorten the course of the illness in Kari but will reduce infectivity and eliminate carriage of the Bordatella pertussis organism in family members. Hospitalisation and isolation are unnecessary unless the clinical condition of the patient warrants inpatient management or in infants less than 6 months of age. Catch-up vaccination should be addressed, but is not the most immediate concern here. There is no requirement to report the family to authorities if they are conscientious objectors to immunisation.

Question 27 Which of the following statements regarding nephroblastoma (Wilms tumour) is CORRECT? a) It usually presents as an asymptomatic abdominal mass b) It is frequently associated with congenital abnormalities c) Most patients diagnosis

will

be

hypertensive

at

d) Prognosis is worse for stage 1 than for stage 4 tumours e) The treatment of choice chemotherapy prior to surgery

involves

Answer: A Nephroblastoma is a tumour arising from the kidney and is a common tumour in children. Most nephroblastomas present as an asymptomatic abdominal mass. Fever and haematuria and hypertension are present in 20 to 25% of patients. The majority of Wilms tumours occur sporadically, although rarely there may be associated malformations and syndromes. Prognosis is worse for stage 4 than for stage 1 tumours. Wilms tumours are first resected, then chemotherapy or radiotherapy may be administered depending on tumour histology and stage.

Question 28 Bobby presents with a pathological in nature. osteosarcoma. Which regarding osteosarcoma

fracture which is found to be Further investigation confirms of the following statements is CORRECT?

sequelae of this common condition. Bathing the lesions to remove the crusts may be helpful.

Question 29

a) The peak occurrence of osteosarcoma is in early childhood

Which of the following statements is CORRECT? Type 1 Diabetes Mellitus:

b) The tumour usually arises in the midshaft of the tibia

a) Is less common than Type 2 diabetes in adolescents

c) The X-ray appearance of osteosarcoma is characteristic

b) Is due to immunological pancreatic Alpha cells

d) Osteosarcoma accounts for 10% of primary malignant bone tumours of childhood

c) Occurs in 6% of siblings of an affected person

e) Osteosarcoma rarely metastasizes to lung

d) Presents with polyuria, polydipsia and weight gain e) Can be managed initially hypoglycaemic agents and exercise

Answer: C The x-ray appearance of osteosarcoma is quite characteristic with destruction of the normal bony trabecular pattern and periostial new bone formation with lifting of the bony cortex to create a Codman triangle. However, a tissue sample is required for diagnosis. Osteosarcoma accounts for 60% of primary malignant bone tumours in childhood, and occurs mostly in adolescents and young adults. More than 40% of tumours arise in the distal femur. At diagnosis a chest CT is essential to look for lung metastases which may be present in 20% of cases, worsening prognosis.

damage

with

to

oral

Answer: C Type 1 diabetes mellitus is the most common type of diabetes in people under 40 years of age, including adolescents. Type 1A diabetes mellitus, or immunemediated diabetes, results from immunologic damage to the insulin-producing Beta cells of the pancreatic islets. About 6% of siblings of an affected person also develop Type 1 diabetes. The classic presentation is with symptoms of polyuria, polydipsia and weight loss. Insulin is the required treatment for Type 1 diabetes.

Question 29 Esther is 7 years old. She presents with a large yellow crusted lesion on her left cheek and similar yellow crusted lesions along her left lower jawline. She has no lesions or rash elsewhere and is otherwise well. Which is the MOST ACCURATE statement regarding this condition? a) Herpes organism

simplex

is

the

likely

causative

b) It is important not to disturb the crusts c) Esther should be screened for immune deficiency d) Topical treatment

mupirocin

is

an

Question 31 This pure tone audiogram is recorded from a 12 year old Maori girl complaining of deafness in her right ear. The MOST likely explanation for this problem is: a) Debris in the external auditory meatus b) Cholesteatoma c) Middle ear effusion d) Toxin-induced nerve damage e) Necrosis of the ossicular chain

appropriate

e) Oral antibiotics should be commenced as early as possible to prevent septicaemia developing

Answer: D The most likely diagnosis is impetigo, with the ruptured vesicles that form yellow crusts and weeping erosions being quite typical of the lesions. Herpes simplex has a different clinical presentation. In childhood, primary HSV infection usually presents as severe acute gingivostomatitis. Impetigo is a very common, highly contagious infection, and does not suggest an underlying immune deficiency. The usual pathogen is Staphylococcus aureus, or Streptococcus pyogenes. For mild or localised impetigo, topical mupirocin 2% ointment or cream 3 times daily for 7 days is appropriate treatment. The lesions must be covered. Whilst oral antibiotics may be indicated for more widespread infection, septicaemia is not a usual

Answer: D The pure tone audiogram provides measurement of the threshold of hearing at a variety of frequencies, by air and bone conduction. The pattern of hearing loss shown on this audiogram is of significant hearing deficit in the higher frequencies, in the right ear. Air and bone conduction are equally affected. This is the pattern of sensorineural deafness. The left ear shows a normal pattern. Toxin-induced nerve damage is the only option which would produce sensorineural hearing loss. All of the others would give rise to a conductive deafness, where the loss would be in the lower frequencies during air conduction. The bone conduction curve would be normal.

Question 32

Answer: C

One minute after birth an infant shows deep cyanosis of the trunk and limbs, makes no reaction to a catheter inserted into the nose, is limp but takes an occasional gasp. What is the Apgar score?

Heberden's nodes are due to primary osteoarthritis. They are firm swellings composed of bone and cartilage on the dorsomedial and dorsolateral aspects of the distal interphalangeal joints in the hand joints. Rheumatoid arthritis is characterised by symmetrical joint involvement (usually proximal interphalangeal joints and metacarpophalangeal joints), morning stiffness greater than an hour, synovial inflammation. Rheumatoid nodules can occur on extensor surfaces of joints in 2030% of people with rheumatoid arthritis.

a) 0 b) 1 c) 2 d) 3 e) Insufficient data

Question 35

Answer: E The table below shows the data required to determine an Apgar score. The scenario given lacks information about the heart rate. Other data given are compatible with a score of 0.

Herman, a 58 year old businessman, finds it difficult to travel by air or train. When he has no option but to travel, he finds the journey very difficult and gets out of the aircraft or train as soon as possible. Because of this he mostly avoids travel and this liability is interfering with his work. What is the MOST LIKELY diagnosis? a) Social phobia b) Agoraphobia c) Depression d) Generalised anxiety disorder e) Panic disorder

Answer: B Agoraphobia is anxiety about being placed in crowded situations from which escape might be difficult or embarrassing, e.g. on aircraft or trains. Generalised anxiety disorder, panic disorder and social phobia are other varieties of anxiety disorders. Anxiety is frequently a symptom of clinical depression.

Question 33 In Huntington's disease the mode of inheritance is: a) X (or sex) linked b) Recessive

Question 36

c) Dominant

In which of the following conditions does acute arthritis commonly occur?

d) Isolated genetic mutation

a) Rubella

e) None of the above

b) Influenza c) Measles

Answer: C Huntington's disease is an autosomal dominant condition with full penetrance. Therefore, the child of an affected parent has a 50% chance of developing the disease. Onset is usually in middle age.

d) Infectious mononucleosis e) Varicella

Answer: A Question 34 Each of the following is rheumatoid arthritis EXCEPT:

characteristic

of

adult

a) Morning stiffness of joints b) Soft tissue swellings c) Herberden's nodes d) Subcutaneous nodules at pressure points e) Symmetrical joint involvement

Acute polyarthritis may occur in rubella, especially in young women. The pain and swelling involve wrists, fingers and knees. It is most marked during the period of the rash, but can persist for up to 14 days after other manifestations have disappeared. Recurrent joint symptoms up to a year have been recorded. Acute polyarthritis is not one of the usual manifestations or complications of the other diseases listed.

Question 37

Question 39

All of the following characteristics describe the 'ideal' vaccine, EXCEPT?

Sanjay, a 45-year-old man, presents with a one-year history of progressive ankle swelling, difficulty with speaking due to an enlarged tongue, pain and paraesthesiae in the forearm and hand, particularly at night, and easy bruising. What is the MOST LIKELY diagnosis?

a) It is heat stable b) It provides lifelong immunity with a single dose c) It has minimal adverse reactions d) It has a good antibody response in the presence of other antigens e) It is administered orally

a) Membranous nephritis b) Allergic reaction c) Chronic leukaemia d) Amyloidosis e) Hypothyroidism

Answer: E All the characteristics listed in the options are desirable in an 'ideal' vaccine, except (e). Each vaccine has an appropriate route of administration which determines its efficacy and probability of side effects. For compliance and ease of administration, delivery without a hypodermic syringe would be ideal but most vaccines are ineffective via the oral route. Other desirable features of the 'ideal' include: being able to combine readily with other antigens, ease of administration and low cost. Although characteristics of the ideal vaccine are well established, developing and producing them is often difficult.

Question 38 Lulu is a three year old child who has swallowed kerosene and is brought immediately to the hospital casualty department. Which of the following measures should be undertaken in the immediate management of Lulu's problem?

Answer: D Amyloidosis involves the deposition of excess amounts of insoluble, fibrous amyloid protein in the extracellular spaces of organs and tissues. The causes are multiple, including neoplastic disease, inflammatory disease and advancing age. There is a rare hereditary form. All organ systems can be involved, with cardiomyopathy, macroglossia and vessel disease. The other conditions can produce oedema, bruising and abnormal hormone production, but not the complete clinical picture as described above. Amyloidosis is usually well advanced by the time it is diagnosed.

Question 40 Amelie is a severely depressed 29-year-old woman with suicidal thoughts who is two months pregnant. Which of the following approaches to treatment would be MOST appropriate?

a) Gastric lavage

a) Counselling and reassurance

b) An emetic

b) Termination of pregnancy

c) Chest x-ray

c) Treatment in hospital oxidase inhibitor (MAOI)

d) Intravenous saline e) Methicillin

d) Treatment antidepressant

at

home

with

monoamine

with

tricyclic

e) Electroconvulsive therapy (E.C.T) in hospital Answer: C Kerosene is an aliphatic, highly volatile hydrocarbon which is poorly absorbed from the gastrointestinal tract. Pneumonitis through aspiration of fumes is the predominant toxic mechanism in children and respiratory distress can be severe and occur rapidly. While a chest xray is not useful for the prediction of lung involvement, serial chest xrays are important to monitor progression. In children who present with lethargy, fever or respiratory signs in the first 4 hours 80% develop pneumonitis. Gastrointestinal irritation is common with nausea and vomiting. There may also be a high fever within 30 minutes of ingestion. Management should be conservative and decontamination (emesis or gastric lavage) should not be attempted - it merely increases the risk of aspiration, and development of pneumonitis. IV saline may be required if haemolysis from the kerosene occurs and hypotension develops.

Answer: E In severely depressed pregnant women ECT has been shown to be safe and effective both antenatally and post partum. Tricyclic antidepressants have been used for over 40 years and are a good choice in a supervised setting but not if the woman is suicidal because of the risk of overdose. MAOIs have not been shown to be safe in pregnancy.

Block 5 Question 1 Audrey is a 61 year old widow who lives alone. She has become very anxious about leaving her house to go shopping, or to attend appointments, like visits to the doctor, since viewing a TV new story about the rise in daytime home burglary. She finds that she has to check and recheck that she has closed and locked all windows and doors over and over again, before she can reduce her anxiety enough to leave her house. This usually takes more than an hour. Some weeks, she does not go out at all, because she still feels anxious after this extensive checking procedure. In such cases her daughter does her shopping for her. Audrey does not have anxiety about other things. What is the most likely diagnosis? a) Generalised anxiety disorder b) Obsessive compulsive disorder c) Posttraumatic stress disorder

(50% of cases) and a fluctuating level of consciousness occurs in 35% of cases. Headaches, localised neurological symptoms and a change of personality may also occur.

Question 3 Jane (age 28) and her husband, Mike, have been trying to have a child for 18 months. Mike has one child with a previous partner. Jane's menses started at age 12 and they have always been infrequent, irregular and sometimes very heavy. Jane used the combined oral contraceptive pill (Diane 35) for 10 years but stopped all contraceptives 2 years ago. Recently Jane has been trying to lose weight. At a height of 165cms, she weighs 85kg. On examination Jane appears normal but she relies heavily on waxing to remove embarrassing facial and lower abdominal hair. What is the MOST LIKELY diagnosis?

d) Agoraphobia

a) Endometriosis

e) Panic disorder

b) Post-pill infertility c) Polycystic ovarian syndrome

Answer: B

d) Pituitary prolactinoma

Audrey displays the features of obsessive compulsive disorder. This is characterised by (a) obsessive thoughts and/or compulsive behaviour that impair everyday functioning, e.g. fears of contamination by germs, repeated handwashing, checking windows and doors etc; (b) the disruptive behaviours are undertaken to relieve the anxiety, and (c) they take up more than one hour per day. The other conditions listed are also forms of anxiety disorders.

e) Hypothyroidism

Question 2 Mavis is 82 years old and recently she fell, landing face down on the floor. She was very shaken and had bruising about her lower face. Two weeks later her family started to notice that Mavis seemed very withdrawn and was sleeping a lot more than usual. Mavis would spend the whole day in bed and she was not really herself. When her family visited, she was increasingly abrupt and moody. Mavis was usually very gentle and quietly spoken. What is the MOST LIKELY diagnosis? a) Subdural haemorrhage b) Extradural haemorrhage c) Dementia d) Stroke e) TIA

Answer: C Polycystic ovarian syndrome (PCOS) is characterized by oligoamenorrhoea, hirsutism, acne, infertility, obesity and insulin resistance. Menarche occurs at the usual time and androgen excess becomes apparent during puberty with development and persistence of hirsutism and/or acne. Diagnosis is largely based on clinical evaluation. Endometriosis is more associated with dysmenorrhoea than irregular cycles. Prolactinoma and hypothyroidism may cause oligoamenorrhoea but not androgen excess. Prolonged use of the combined oral contraceptive pill is not associated with infertility after the cessation of its use.

Question 4 Mr Davy has had severe intermittent pain in the right side of his back, radiating into his right groin and to the tip of his penis. It has been present for the last 6 hours. He feels continuously nauseous, and with every spasm of pain, he feels he cannot lie still but must move around. Sometimes, curling himself into a tight ball helps. He has had one similar, but less severe episode of pain one year ago that resolved spontaneously. On examination he is afebrile and his urine has only a trace of red blood cells. What is the MOST LIKELY diagnosis? a) Appendicitis

Answer: A Subdural haemorrhage may be insidious in onset, and the elderly are particularly susceptible due to brain shrinkage. A history of trauma may not be recalled

b) Urinary tract infection c) Pyelonephritis d) Ureteric calculi e) Diverticular disease

Answer: D Renal calculi (stones) may be asymptomatic. However calculi in the ureters commonly cause pain from the loin, into the groin and/or pain in the tip of the penis. There is usually no penile redness or discharge and few other abdominal signs are present, unless urinary obstruction is occurring with urethral calculi. Haematuria and loin tenderness are common. Question 5 John is a 28 year old unemployed man with multiple complaints, including headache, low backache, upper abdominal pain, pain in both feet, nausea, bloating, impotence and weakness in both forearms and left leg. Physical examination shows no abnormal clinical signs. Previous investigations including chest X-ray, full blood count, biochemical profile and abdominal ultrasound show no abnormality. What is the MOST LIKELY diagnosis? a) Factitious illness b) Munchausen's syndrome c) Conversion disorder d) Hypochondriasis e) Somatisation disorder

Answer: E In somatisation disorder the patient has multiple physical complaints referable to different organ systems, including at least four pain, two gastrointestinal, one sexual and one pseudoneurological symptom(s) which are not consistent with any specific diagnosis. There is significant impairment of social, occupational or other important area of functioning. Treatment involves behaviour modification and limitation of further investigations.

Question 6 Myra, a 38 year old bank teller, presents with a painful right lower leg. On examination, she has some dilated, tortuous veins mostly on the posterior and lateral aspects of her calf. There is an area of redness and heat over one of these veins, and a firm cord like lump in the vein, 3 cm long, which is tender to touch. The MOST correct statement is: a) Myra requires antibiotic treatment with flucloxacillin b) There is a small risk of extension into deep veins c) Myra should have subcutaneous low molecular weight heparin while awaiting a venous Doppler scan

extension is suspected it would be preferable to confirm this by doppler ultrasound before commencing anticoagulants. Varicose veins have many risk factors, one of which is prolonged standing. This increases hydrostatic pressure leading to chronic venous distension and secondary valvular incompetence. Women are particularly susceptible as the vein walls become more distensible under the cyclic influence of progesterone. The condition is likely to resolve spontaneously over a few days. Non-steroidal antiinflammatory agents may be used to reduce pain and local inflammation, and graduated compression stockings may be helpful if the condition does not resolve quickly. Thrombophlebitis is not usually infective however antibiotics may be used in the case of persistent or severe symptoms.

Question 7 Little Andrew, aged 18months, was 'helping' Dad in the shed, when he began screaming and rubbing his eyes. He had climbed onto the workshop bench, on which was kept a variety of potential ocular hazards. Which of the following substances is potentially MOST harmful to Andrew's eyes? a) Methylated spirits b) Superglue c) Acetic acid d) Dog shampoo e) Powdered cement

Answer: E Cement is alkaline, and alkaline burns are more dangerous than those from other chemicals. Alkali has the potential to penetrate the cornea and gain access to the anterior chamber, causing uveitis, secondary glaucoma and cataract. Alcohols and solvents cause severe pain initially but although the epithelium is burnt, it tends to regenerate quickly. Superglue, while it may cause distress in gluing eyelids together, is actually not harmful to the eye- in fact it is sometimes used in treatment of corneal wounds. The weak acid, and the dog shampoo, will both cause more irritation than actual damage. First aid treatment for any substance splashed into an eye is profuse irrigation.

Question 8 Fred is a 74 year old hypertensive man who has been found to have a 55mm fusiform abdominal aortic aneurysm, discovered when he had an abdominal ultrasound for right flank pain two days ago. Of the following, which is NOT a risk factor for rupture of Fred's aneurysm?

d) This condition is unrelated to her occupation

a) Persistently elevated mean arterial pressure

e) The condition is unlikely to resolve without specific treatment

b) The fact cigarettes/day

that

he

still

smokes

c) His chronic obstructive pulmonary disease Answer: B

d) The fact that Fred is male

Myra has superficial thrombophlebitis, a relatively common problem. In this particular site, it is likely to be in the short saphenous vein system and the risk of extension to the deep system via perforating veins is small. However, it is not negligible. If deep vein

e) The size of the aneurysm

15

Answer: D Risk factors for the development of an abdominal aortic aneurysm (AAA) include smoking, increasing age, hypertension, family history, chronic obstructive pulmonary disease (COPD) and being male. Risk factors for AAA rupture are an elevated mean arterial pressure, continuing to smoke, more severe COPD and having an aneurysm that is either rapidly enlarging or is measured at >50mm diameter. Although women have a lower incidence of AAAs which tend to be smaller, they have a much higher risk of rupture. In this scenario, the flank pain may well be an indicator of expansion of the aneurysm.

Question 9 Colin is 22 years old. His right arm was amputated above the elbow when it became caught in the industrial mulcher he was using. His mate tied his own T shirt firmly around the stump and brought him to hospital. On arrival, 15 minutes later, the T shirt is soaked, and blood is trickling out. Colin is pale, his skin is cool and clammy, and he looks anxious. His pulse is 110 beats/min and his BP 130/95 mmHg. His respiratory rate is 20 breaths/min. Capillary refill time is 5 seconds. You are able to insert an intravenous cannula in his left arm. Which fluid orders are MOST appropriate in this circumstance? a) 1 litre Normal saline as a bolus, then 1 Litre 4% dextrose in 1/5N saline b) 2 units O negative blood c) 500ml normal saline

105/70mm Hg. There appears to be diminished excursion of his right chest wall, and the breath sounds are hard to hear on the right. There is hyperresonance to percussion on of the right chest. Your IMMEDIATE response should be? a) Arrange an urgent chest X ray b) Perform intubate

rapid

sequence

induction

and

c) Insert a thoracostomy tube in the right fifth intercostal space in the anterior axillary line d) Insert a wide bore needle in the right second intercostal space e) Insert a wide bore needle in the left second intercostal space

Answer: D Brendon has almost certainly developed a right tension pneumothorax, as indicated by his increasing dyspnoea , and the physical signs described above. This is a lifethreatening condition which requires urgent management. Decompression with a wide-bore needle in the second intercostal space, in the midclavicular line of the affected side is potentially life-saving, and allows time for the more complex procedure of the tube thoracostomy to follow. Tension pneumothorax is a clinical diagnosis, and emergency treatment should not be delayed for X ray confirmation. Intubation and ventilation may turn a simple pneumothorax into one under tension. It is not indicated in this situation.

d) 500ml colloid e) 1.5 L Normal saline

Answer: E Colin is a young adult, apparently fit. His signs indicate that he has suffered a class 2 haemorrhage, and has lost approximately 15-30% of his total blood volume or 750-1500ml. So far his body has compensated well, but this may not be sustained. He requires replacement of volume and the most commonly recommended fluid is an isotonic crystalloid such as normal saline. O negative blood is not required in this situation. There would normally be time to obtain cross matched blood if bleeding could not be controlled. Hypotonic saline/dextrose solutions are not appropriate. These fluids are used to maintain fluid balance in a normovolaemic, normonatraemic patient and do not restore intravascular volume in the volume-depleted patient.500ml of normal saline is not sufficient. While there are some theoretical advantages to using colloid as the replacement fluid, there is little evidence of improved outcome from using this instead of crystalloid. 500ml of colloid is not sufficient on its own. 1-2 litres as the initial bolus, for an adult of average build is appropriate in this circumstance, then the patient's response should be assessed.

Question 10 Brendon is a 35 year old man who has been involved in a motor vehicle accident. He was wearing his seat belt, but it did not hold and he was thrown against the steering wheel. He is anxious and increasingly dyspnoeic. His pulse is 126 beats/minute and his BP

Question 11 Mary, aged 65, had a laparotomy for resection of a bowel cancer seven days ago. She has been progressing well, but has just noticed some pinkish fluid leaking from her wound. Which of the following is TRUE regarding this situation? a) This complication occurs in 10% of older patients undergoing abdominal surgery b) The wound will require urgent surgical repair c) There is a mortality rate of 1% associated with this complication d) If the wound breaks down, it must heal by secondary intention e) The appropriate management is intravenous antibiotics

Answer: B The serosanguinous discharge heralds dehiscence of the wound, and after undertaking any necessary resuscitation and preparations for theatre, Mary should return to theatre as soon as possible. Early wound dehiscence is a serious complication, usually occurring around the 7th to 10th post-operative day. It occurs in fewer than 1% of laparotomy wounds but can have a mortality of around 30%. Risk factors include poor nutritional state, malignancy, obesity, prolonged surgery, infection or coughing. The wound cannot be left to heal by secondary intention. Intravenous antibiotics may form part of the management but will not suffice alone.

Question 12

Answer: A

Kevin, a 45 year old labourer, had a laparotomy five years ago when he suffered a ruptured appendix. He has recently noticed a dragging sensation in the region of his scar, especially when lifting heavy objects at work, and now presents with a swelling of 1.5 cm diameter in the medial end of his scar. Concerning Kevin's problem, which of the following is TRUE?

As increasing age is a risk for colorectal cancer, a patient over the age of 40 who presents with PR bleeding should have a digital rectal examination and be investigated by colonoscopy. If this is not available a flexible sigmoidoscopy and double contrast barium enema would be satisfactory. Rectal bleeding is a common symptom of haemorrhoids, but a rectal neoplasm may also cause PR bleeding. Even in the presence of obvious haemorrhoids patients at increased risk for colorectal cancer should be investigated. FOBT is a screening test , not a diagnostic investigation.

a) Kevin's lean, muscular predisposes him to this problem

body

type

b) The fact that the scar is paramedian and in the lower abdomen predisposes to this problem c) The problem is of nuisance value only, as only fatty tissue protrudes into the swelling d) Kevin should have surgical repair as soon as convenient e) Kevin should wear an abdominal support garment to prevent complications

Question 14 Florence, aged 50, has decided to have a haemorrhoidectomy after months of unsuccessful conservative management of her haemorrhoids. In obtaining informed consent, you discuss with her the potential complications of haemorrhoidectomy. Which of the following is the LEAST likely complication? a) Urinary retention

Answer: D Kevin has an incisional hernia, which is a protrusion of abdominal contents into the subcutaneous plane through a defect at the site of a previous incision. Incisional herniae should be repaired as soon as convenient because they can increase in size over time and may become very difficult to repair. More particularly, as with most herniae, they may become irreducible, with possible obstruction and strangulation of abdominal contents including bowel. Incisional herniae are more common in obese patients in whom there is fatty infiltration of the tissues, increased intra- abdominal pressure and reduced muscle tone. They are more common in midline and upper abdominal scars. There is no evidence that any supportive garment will prevent complications in an incisional hernia although it may relieve discomfort.

b) Post-operative bleeding c) Sepsis d) Faecal incontinence e) Pain Answer: C Sepsis is fortunately a very rare complication of hemorrhoidectomy. Urinary retention occurs in approximately 5-10% of cases and may be due to spinal anaesthesia and/or the use of IV fluids and urinary catheter intraoperatively. Bleeding is uncommon but may be severe. It can occur in the first 24 hours or 7 to 10 days later due to local infection. Pain is fairly common and may be severe. It is associated with faecal impaction and incontinence. Later rare complications include fissures, fistulae and anal stenosis.

Question 13 Peter is 47 years of age and presents with a single episode of bright red bleeding per rectum (PR). which he noticed after passing a bowel motion this morning. He is unaware of any significant family history of colorectal problems. On examination Peter has some obvious haemorrhoids but nothing else of note on rectal or proctoscope examinations. What is the MOST appropriate advice for Peter? a) In view of his age he should have a colonoscopy to investigate this bleeding b) As there is an obvious cause for his bleeding, no further investigation is needed at present c) As he has no significant family history of colorectal disease, he only needs reassurance d) Monitoring with 6 monthly faecal occult blood testing (FOBT) is required e) He should have a trial of increased fibre in his diet and review the haemorrhoids in 3 months

Question 15 Brian, a 52 year old man, walks awkwardly into your rooms. He complains of severe pain, which he indicates as being quite deep in his rectum. He says the pain began earlier in the day but has become much worse in the last hour and he it feels like 'something coming down' in his back passage. Which of the following statements MOST accurately describes Brian's condition? a) Brian has haemorrhoid

a

thrombosed

external

b) Brian's deep pain is due to prolapsing internal haemorrhoids c) Brian has a rectal prolapse d) Brian has grade three haemorrhoids e) Brian haemorrhoids

has

strangulated

internal

Answer: E

Answer: D

The pain from strangulated internal haemorrhoids is typically felt as a deep pain. Prolapsing internal haemorrhoids can cause perianal pain by causing a spasm of the anal sphincter complex. If the haemorrhoids become trapped by the spasm, they become engorged with secondary venous and later arterial thrombosis, and become irreducible. This is known as 'strangulation' and results in deep seated pain, especially if necrosis and ulceration occur. The pain of thrombosed external haemorrhoids is felt perianally. Rectal prolapse is rarely painful. Brian's haemorrhoids are now irreducible, so are no longer grade 3 (require manual reduction).

Unilateral renal agenesis is not uncommon and the solitary kidney compensates by hypertrophy and maintains normal renal function. It is usually accompanied by ureteral agenesis. Potter's syndrome is bilateral renal agenesis and it is fatal.

Question 16 Jason is a 30 year old mature age medical student. He has been hospitalised following a haematemesis due to a Mallory-Weiss tear. Jason asks for an explanation about Mallory-Weiss tears. Which of the following statements is FALSE? a) Mallory-Weiss tears are tears in the mucosa of the lower oesophagus b) Haematemesis in Mallory-Weiss tears is always preceded by retching or vomiting c) Bleeding from Mallory-Weiss tears stops spontaneously in 80-90% of patients

Question 19 Jane is 45 years of age and she has noticed the following changes in herself over the last 4 months. She has lost weight, her eyes feel dry, but they are constantly watering and she feels irritable and 'on edge' and occasionally experiences palpitations. Her periods have become irregular, her hair is thinning and her fingernails seem very brittle. Her father and older sister experienced the same symptoms when they were 40 years of age. What is the MOST LIKELY diagnosis? a) Graves' disease b) Toxic adenoma c) Simple diffuse goitre d) Multi-nodular goitre e) Hashimoto's thyroiditis

Answer: A

The classical presentation of Mallory-Weiss syndrome is haematemesis from a tear in the oesophagus, brought on by prolonged vomiting of any cause. It is often associated with alcoholic excess but this is NOT always the case. Haematemesis may occur without prior retching or vomiting. The tear is typically a longitudinal one in the mucosa of the lower oesophagus close to the gastro-oesophageal junction. The bleeding settles spontaneously in 80-90% of cases of Mallory-Weiss tears. Not all MW tears present with haematemesis. In a small proportion, melaena, haematochezia, syncope or abdominal pain are the presenting symptoms.

Graves' disease is characterized by hyperthyroidism and one or more of the following: goitre, exophthalmos, and pretibial myxoedema. It is an auto-immune disorder that has a genetic component and commonly presents in women aged 40 -50 years. Toxic adenoma can occur at any age.It usually presents as a single thyroid nodule not a goitre, and hyperthyroidism. Simple diffuse goitre occurs mostly in younger women aged 15-25 years. The thyroid gland is enlarged but the person is euthyroid. Multi-nodular goitre is often a simple diffuse goitre that has progressed as the person has become 'middle-aged' or elderly. The goitre is 'lumpy', not diffusely enlarged and initially the person is euthyroid but may become hyperthyroid in the long-term. Sometimes it causes difficulty with swallowing and breathing if large. Hashimoto's thyroiditis is a chronic inflammation of the thyroid caused by autoimmune factors. It causes painless enlargement of the thyroid gland or fullness in the throat and many patients have hypothyroidism when first seen. Other forms of autoimmune disease are common.

Question 18

Question 20

Hugo did not realise until he volunteered to be a kidney donor that he had been born with only one kidney. Which ONE of the following statements is TRUE?

Amanda, 47 years, has noticed her right eyelid is higher than her left and her right eye seems more prominent. She first noticed she can apply her mascara to her left eyelashes easier if she tilts her head back and looks upward. Her contact lenses still fit perfectly. Amanda is otherwise well with no other symptoms or signs. What is the MOST LIKELY diagnosis?

d) Alcoholic binge drinking may be associated with Mallory-Weiss tears e) Haematemesis is not a universal symptom of a Mallory Weiss tear

Answer: B

a) Hugo's syndrome

condition

is

known

as

Potter's

b) Unilateral renal agenesis is uncommon c) Usually in unilateral renal agenesis there are still two ureters d) In unilateral renal agenesis the solitary kidney maintains normal renal function e) Hugo needs an annual ultrasound scan of his solitary kidney

a) Bell's palsy b) Hyperthyroidism c) Myasthenia gravis d) Horner's syndrome e) Optic nerve glioma

Answer: D Ptosis is drooping of the upper eyelid associated with an inability to elevate the lid completely. Nerves from the sympathetic chain innervate the superior tarsal muscle causing unilateral partial ptosis that can be overcome by looking upward. Horner's syndrome includes unilateral partial ptosis, ipsilateral constricted pupil and ipsilateral lack of sweating of the face. Myasthenia gravis usually causes bilateral partial ptosis. Hyperthyroidism causes protruding eyes (proptosis/ exophthalmos) which may be unilateral. Bell's palsy (VII nerve paralysis) prevents the patient from forcefully closing their eyes and they have bilateral wide palpebral fissures. Optic nerve glioma causes painless progressive proptosis.

Question 21 Michelle needs a transfusion after a major motor vehicle accident. In the accident her pelvis was fractured, both femurs have mid-shaft fractures and she sustained a hemothorax requiring a chest drain. Michelle was trapped for an hour before the fire rescue could cut her out of her vehicle. Michele has blood group O Rh positive. Which of the following statements is TRUE? a) Michelle has type A antigens on her red blood cells b) Naturally occurring A and B antigens are called isoagglutinins

shallow respiration, no abnormal physical findings. Of the following, which is the MOST LIKELY diagnosis? a) Agoraphobia b) Posttraumatic stress disorder c) Generalised anxiety disorder d) Panic disorder e) Acute psychosis

Answer: D Karen's story displays the features of a panic attack which is the cardinal manifestation of panic disorder. Patients with panic disorder experience repeated unexpected attacks of intense, disabling anxiety. In between attacks they experience at least one month of worry about having further attacks and/or fear of losing control, going mad or dying. Agoraphobia is an irrational fear of being trapped in a place from which escape is impossible. Patients with posttraumatic stress disorder are repeatedly distressed by re-experiencing highly traumatic events. Generalised anxiety disorder involves persistent excessive and/or unrealistic worry accompanied by other signs and symptoms, such as muscle tension, restlessness and feeling on edge. Acute psychosis is a severe mental disturbance involving hallucinations and/or delusions.

c) Michelle has anti-A and anti-B antibodies d) Persons with Type O blood are "universal recipients" e) Michelle lacks the D antigen

Answer: C The ABO blood group system is the most important in transfusions. Persons with Type O blood are "universal donors" because their red blood cells lack A or B antigens. Type O individuals produce their own anti-A and anti-B. However, their cells are not recognised by any naturally occurring anti-A or anti-B antibodies (otherwise known as isoagglutinins), when their red blood cells are transfused. The Rh system is the second most important blood group system in pretransfusion testing. Rh 'positive' individuals have the D antigen of the Rh system, while people lacking the D antigen are Rh 'negative'.

Question 23 Sue is 30 years old and concerned she will get breast cancer because her mother had breast cancer diagnosed when she was 45 years of age. Which of the following statements is TRUE regarding breast cancer? a) BRCA1 and BRCA2 mutations account for 60% of breast cancer cases b) Sue does not have an increased risk of breast cancer because her mother had breast cancer c) Breast cancer is a disease of younger women d) Sue should have bilateral mastectomies to prevent breast cancer developing e) If Sue does develop breast cancer she is most likely to develop it after she is 50 years of age

Question 22

Answer: E

Karen is a 21 year old university student. She is accompanied to your consulting rooms by two women friends who observed her collapse this morning in a bathroom of their university residence. Karen tells you that soon after she woke today she suddenly began feeling unwell, with intense fear, palpitations, sweating, shortness of breath, nausea and tingling in her fingers. . She does not remember anything after entering the bathroom. This is the third similar attack that she has experienced during the past two months. After the first attack, she worried that she 'might be going crazy' but postponed seeking medical advice for fear of being institutionalised, like one of her aunts who has schizophrenia. She does not smoke, drink alcohol or use illicit drugs. The only medication she takes is paracetamol occasionally for headaches. On examination you find: PR 85/min, BP 135/95 mm Hg, moist palms,

Although mutations in BRCA1 and BRCA2 are associated with an increased risk of breast cancer, and the lifetime risk of developing breast cancer in women who have these mutations approaches 80%, these lesions account together for less than 10% of breast cancer cases. Women who have first-degree relatives who have developed breast cancer do have an increased risk of developing breast cancer themselves, and if their firstdegree relative with breast cancer was diagnosed before age 50 they have a higher risk of developing breast cancer than women whose first-degree relative was diagnosed after age 50. However, in all cases, breast cancer is uncommon in young women. Furthermore, most women with affected first-degree relatives with breast cancer who themselves develop breast cancer do so after 50 years of age In the absence of mutations in BRCA1 or BRCA2, the risk associated with a positive

family history does not seem of sufficient magnitude to justify routine bilateral mastectomy.

Question 24 At birth Sammy has a cleft lip but otherwise looks normal. Sammy's parents are very distressed about this, and are concerned to know if Sammy has anything else wrong which they cannot see yet, or that may develop when he's older. Which of the following is TRUE? a) Sammy is likely to have Pierre Robin syndrome and a cleft lip is just part of this syndrome b) Sammy has a cleft lip due to his mother's use of antidepressants c) Sammy is likely to have this isolated abnormality and no other problems except the cleft lip d) Sammy is likely to have congenital dislocation of the hips as well as his cleft lip e) Cleft lip and cleft palate are associated with talipes (clubfoot deformities)

Answer: C The cleft may vary from involvement of the soft palate only, to a complete cleft of the soft and hard palates, the alveolar process of the maxilla, and the lip. The mildest form is a bifid uvula. These children have normal intelligence and development. Cleft lip with or without cleft palate occurs in 1:700-1000 live births, more often in Asian groups and less often in African Americans; more often in males. Cleft palate alone occurs in 1:2000 across all races with slightly more females affected. There may be genetic and environmental factors including maternal smoking and use of alcohol, retinoic acid and anticonvulsants. Associated anomalies occur in about 15 to 20% of cases of cleft lip with or without cleft palate but in 50% of cases of cleft palate alone. Pierre Robin syndrome typically presents with micrognathia (small mandible) and a cleft soft palate. Congenital dislocation of the hip seems to be secondary to laxity of the ligaments around the hip or to in utero positioning. Clubfoot (talipes) deformities, result in the foot being plantar flexed, inverted, and markedly adducted. Neither of these congenital abnormalities is associated with cleft lip or palate.

Question 25 Mandy has had migraines since she was a teenager. They are the classical migraine with a prodrome when she is clumsy, yawns a lot, is tired, has a stiff neck and feels irritable. Then she gets the aura, with 'sparks' in her vision. Then she gets a severe headache that starts at the back of her neck and moves to one of her temple areas and then her forehead. She feels sick and wants to curl up in bed, in a dark room, and let the headaches pass, which it usually does in about 6 hours. Which of the following statements is TRUE regarding migraine headaches? a) Migraine headaches are equally common in women and men b) In Australia 30% of the population have migraine headaches

c) Migraine is rare in children less than 10 years of age d) The most common form of migraines has a prodrome and an aura e) Migraine is accompanied by nausea in 90%, vomiting in 60% and diarrhoea in 15% of attacks

Answer: E There are two main types of migraine: classical migraine (migraine with aura) and common migraine (migraine without aura), the latter accounting for the majority of migraine headaches. About 10% of the population in Australia have migraine. Migraine usually starts during the teenage years or early adult life and occurs more commonly in women than men (ratio 3:1). In children the incidence is 3-7%. Migraine may be accompanied by a variety of symptoms other than the typical nausea, vomiting and photophobia

Question 26 Dimitri is a 45 year old man who presents with insomnia. He goes to sleep at night without difficulty, but wakes frequently from distressing dreams in which he is being forced to watch people being tortured. Then he has great difficulty in going back to sleep. Dimitri has been in Australia for two years, having emigrated from Kosovo, where he was imprisoned for a year. His wife says he had a 'bad experience' while in detention, but will not talk about it. Over the past six weeks, he has become irritable, prone to outbursts of anger and has begun drinking heavily. This has led to marked tension in the home and Dimitri's workplace. Which of the following is the probable cause of Dimitri's distress? a) Acute stress disorder b) Posttraumatic stress disorder c) Panic disorder d) Phobic disorder e) Generalised anxiety disorder

Answer: B Posttraumatic stress disorder is an anxiety disorder of more than one month's duration, consequent upon a severe traumatic experience in the individual's past, and which s/he now re-experiences in one or more ways (e.g. flashbacks or dreams). This is accompanied by avoidance of stimuli which recall the event, numbing of the individual's responsiveness, symptoms of arousal (e.g. insomnia) and distress or social/occupational impairment.(see refs for full diagnostic criteria). In contrast , an acute stress disorder develops soon after the traumatic experience. A panic attack is the cardinal manifestation of panic disorder. Patients experience intense, disabling anxiety and may fear they are losing control, going mad or dying. Generalised anxiety disorder involves persistent excessive and/or unrealistic worry, accompanied by other signs and symptoms, such as muscle tension, restlessness and feeling on edge. Patients with phobic disorders display marked fear of objects or situations which provoke an immediate anxiety reaction.

Question 27 Jane is 24 years of age and 10 weeks pregnant with her first child. She has just been diagnosed with her first ever urinary tract infection. Which drug would you choose to treat Jane's urinary tract infection? a) Trimethoprim b) Cephalexin

sympathetic overactivity or cerebral compromise, resulting from hypoglycaemia, rapidly progress to coma, if untreated. Hypoglycaemic coma commonly occurs in well-controlled diabetic patients, and is due to their diabetic medications eg: longer acting sulphonylureas. However blood glucose should always be tested (dipstick and laboratory confirmation) in an unconscious patient (diabetic or not) and hypoglycaemia assumed to be the cause of any coma, until proven otherwise.

c) Amoxycillin d) Norfloxacin e) Erythromycin

Answer: B The important time for teratogenic effects of drugs given in pregnancy is in the first trimester. All drugs, if possible should be avoided in the first 12 weeks of pregnancy. However if Jane has a urinary tract infection she requires treatment. Trimethoprim and norfloxacin (usually used to treat pyelonephritis) are category B3 drugs in pregnancy and should be avoided. Amoxycillin and Cephalexin are both category A in pregnancy, however amoxycillin is only recommended if susceptibility of the organism is proven. Erythromycin is also category A but unsuitable in the management of urinary tract infections.

Question 28 May was found at home in a coma and brought into hospital, where she is now recovering well. May is 80 years of age, and she has been well most of her life, but in the last 5 years she has gained about 10kg in weight. During the past week or two, before she was brought into hospital, May has been tired, sleepy, 'dry as a chip', forever running to the toilet to pass urine, and yet she had been unable to drink enough to satisfy her thirst. What was the MOST LIKELY diagnosis when May was brought into hospital? a) Diabetic ketoacidotic coma b) Hypoglycaemic coma c) CVA with coma

Question 29 Mike has come to you to discuss vasectomy. He is 45 years of age, and he has three children to his current partner Sam. Sam has tried many different contraceptives, but none have been satisfactory. She has finally told Mike he has to do something about contraception for them now they have had all the children they want. Mike is very nervous about any type of surgery, especially if it involves his genital area. He has never been near a surgeon in his life. Which of the following statements is CORRECT? Vasectomy: a) Is not as permanent as male sterilisation b) Is not effective immediately c) Is totally functionally reversible d) May be followed by a reduced testosterone level e) May result in a reduced volume of semen production

Answer: B Vasectomy is sterilisation of the male and it involves a small incision in the scrotal skin under local anaesthetic. The vas deferens is separated from its blood supply and approximately 1cm of it is removed between ligatures. Post-vasectomy it takes up to 3 months for the sperm to be eliminated from the ejaculate (the volume of sperm in the vas deferens between the point of excision and the tip of the penis). Vasectomy is to be considered irreversible as microsurgery may repair the vas deferens, but sperm function may never return ( due to the production of sperm antibodies). There is no change to male testosterone levels, balding patterns or libido.

d) Hyperosmolar non-ketotic coma e) Hypothyroid crisis

Answer: D Hyperosmolar non-ketotic coma (HONC) occurs in elderly patients with Type 2 diabetes mellitus, but the history of diabetes is usually unknown. It has an insidious onset that includes polyuria and polydipsia, severe dehydration, and an impaired level of consciousness, which correlates with plasma osmolality. Coma is usually associated with an osmolality >440mmol/l. Respiration is usually normal. Patients may rarely present with a CVA, seizures or an MI, but the underlying disorder is primarily diabetes. Blood glucose is usually >40mmol/l, there is severe hypernatraemia and dehydration, with a relatively normal arterial pH, unless there is coexisting lactic acidosis. Rehydration and insulin are the mainstays of treatment and causes of infection should be sought as well as ECG changes consistent with infarct or ischaemia. Diabetic ketoacidotic coma only occurs in Type 1 diabetes. Hypoglycaemic coma has more rapid onset than HONC. The preceding symptoms of

Question 30 Pamela, an 18 year old first year music student, complains of disabling anxiety. She says she has always been 'nervous in front of strangers', but her problem has been aggravated since she started her music studies. She feels well during the weekend, but is very anxious during the week, and wonders whether she should withdraw from the course. Her main problem is fear of solo performances. Her tutor requires all students to perform solo each week without prior warning for of a group of staff members . Pamela finds this very unnerving. She cannot think or play properly under these conditions, and has 'frozen' and burst into tears on more than one occasion. Which of the following is the most likely diagnosis? a) Generalised anxiety disorder b) Panic disorder c) Phobic disorder d) Obsessive compulsive disorder e) Posttraumatic stress disorder

Answer: C The features of phobic disorder are (a) a marked persistent fear of objects or situations, exposure to which provokes an immediate anxiety reaction that may take the form of a panic attack; (b) avoidance behaviour to avoid the phobic stimulus; and (c) anxiety is provoked only in specific situations. Pamela has a social phobia, characterised by fear of social or performance situations, where she is exposed to unfamiliar individuals, or to possible evaluation by others. Medication with selective serotonin reuptake inhibitors (SSRIs) may be helpful, but the mainstay of management is behaviourally focussed psychotherapy. Generalised anxiety disorder, panic disorder, obsessive compulsive disorder and posttraumatic stress disorder are other anxiety disorders each with its characteristic presentation.

Question 31 Alison has been taking the tricyclic antidepressant drug (TCAD) amitriptyline for 6 years. She started taking it when her husband John passed away with cancer. Alison is "much better" now, as she has adapted to life without John. She is sleeping well, her appetite has returned, and even though she still desperately misses John, she no longer avoids neighbours and friends, and she feels less like crying every minute of the day. Alison stopped her amitriptyline suddenly last week without consultation with her doctor. Which of the following is NOT common after abrupt cessation of TCADs? a) Cholinergic activation - abdominal cramps, diarrhoea and vomiting b) Sleep disturbance - insomnia and vivid dreams c) Somatic distress - flu-like symptoms and headache d) Cardiovascular symptoms - palpitations and arrhythmias e) Psychiatric symptoms - anxiety and agitation

Answer: D All the other options are withdrawal syndromes associated with withdrawal from tricyclic antidepressant drugs. TCADs can cause adverse effects such as orthostatic hypotension, conduction defects and arrhythmias while they are being used. However upon withdrawal of TCADs cardiovascular symptoms are not common. Withdrawal from benzodiazepines is more likely to be associated with cardiovascular symptoms including palpitations, flushing and hyperventilation.

b) Mandy will develop renal failure within 12 hours c) Mandy should be given oral methionine d) Other commonly prescribed medication taken at the time of a paracetamol overdose will not alter subsequent liver damage e) FFP (fresh frozen plasma) is the treatment of choice when the prothrombin time is abnormal following paracetamol overdose

Answer: A N-acetyl cysteine (iv) is given to all severe paracetamol overdoses (>10grams) presenting with symptoms or abnormal investigations (liver function tests (LFTs), prothrombin time (PT)). All patients with paracetamol plasma levels on or above the "Normal" treatment line (when plasma paracetamol levels are plotted against time in hours), presenting up to 24hours following ingestion, should also be given N-acetyl cysteine. Only patients presenting within 10-12 hours, who are allergic to N-acetyl cysteine, should be given oral methionine. Oliguria and renal failure generally occur late (day 3 following ingestion) following paracetamol overdose. However 10% of patients develop acute renal failure from acute tubular necrosis. Vitamin K, 10mg, given intravenously (iv) is preferable in paracetamol overdose and FFP (fresh frozen plasma) should be avoided, unless there is active bleeding. FFP may make future management, including liver transplant more difficult. Patients on enzyme-inducing drugs (e.g. phenytoin, carbamazepine, rifampicin, phenobarbitone) or those who are malnourished (e.g. anorexia, alcoholism) develop paracetamol toxicity and require intervention at lower plasma paracetamol levels than previously healthy patients on no enzyme-inducing medications.

Question 33 Shamila is a 16 year old schoolgirl who consults you because she is very unhappy at home and says she is considering suicide. You assess her as being clinically depressed. Which ONE of the following strategies would you adopt NEXT to deal with the threat of suicide in this case? a) Refer Shamila to a psychiatrist b) Referral to a local mental health crisis team c) Admit Shamila urgently to the psychiatric ward of the local hospital d) Ask Shamila if she has made any suicidal plans e) Commence cognitive behavioural therapy immediately

Question 32 Mandy ingested 30grams of paracetamol 18 hours ago, and she is slightly nauseous and tearful but otherwise asymptomatic. Mandy is an adult Caucasian female, 65 kg, with no pre-existing illnesses. She is a non-smoker, does not drink alcohol and is on no other medications. She has no known allergies. You ordered some investigations when Mandy arrived at the hospital and they show that she has elevated hepatic transaminases (ALT, AST), prolonged prothrombin time and hypoglycaemia. Which of the following is TRUE? a) Mandy should be given N-acetyl cysteine

Answer: D When patients have suicidal thoughts, the treating doctor should take careful note of the context. Patients who have made definite plans to commit suicide, or who have obtained the means with which to carry it our, e.g. a weapon, are at much greater risk of killing themselves than those who have simply contemplated the matter in theory. In cases where there is serious intent to commit suicide, the patient should be regarded as seriously depressed and referred for urgent specialist attention. How this is achieved will differ in different areas. In

some cases the best approach may be to refer the patient to the local mental health crisis team. Where such a team does not exist, urgent referral to a psychiatrist or urgent admission to a psychiatric facility is indicated.

Question 34 Marty is a 42 year old man who presents complaining of chronic headaches. He says he has come to see you only because his wife insisted. On questioning he is not very informative but admits to having a few beers after work most days. On examination you note his complexion is flushed, there is facial telangiectasia and some periorbital puffiness. His BP is 150/95 mm Hg. You suspect that Marty's problems relate to hazardous drinking. Which ONE of the following strategies would be best for obtaining confirmation of your suspicions? a) Confront Marty outright and demand the truth about his drinking b) Phone Marty's wife while he is with you and ask her about his drinking c) Administer an Alcohol Identification Test (AUDIT)

Use

Disorders

d) Take a blood sample for a carbohydrate deficient transferrin (CDT) test e) Take a blood sample for a blood alcohol concentration (BAC) measurement

Answer: C Patients with a drinking problem often do not openly acknowledge how much they are drinking, so other means have to be employed to determine whether they are drinking hazardously. The best approach is to administer a questionnaire (such as AUDIT or CAGE) which explores the patient's drinking pattern and its potential effects on his/her life. Obtaining corroborative information from family members is also helpful but should not be the main approach to obtaining information. Laboratory tests are also useful but there is a considerable incidence of false negatives. The CDT test is relatively insensitive - it requires a consumption level of 60 or more g of alcohol per day to record a positive result. The BAC will only be positive if the patient has been consuming alcohol during the preceding hours before the test.

Question 35 Myra is an 80 year old woman who is brought to your consulting room by her daughter and son-in-law who are concerned that she may have dementia because of her increasing forgetfulness. Which ONE of the following initial strategies would be best to determine whether Myra may have dementia? a) Take blood to measure thyroid function b) Do a thorough neurological examination c) Do a general physical examination including urinalysis d) Establish rapport and administer the Mini Mental State Examination e) Take a medication history and administer the Alcohol Use Disorders Identification Test

Answer: D The Mini Mental State Examination is the appropriate test to examine the patient's orientation. It will detect cognitive impairment, whether due to dementia, depression or delerium. Differentiating these three conditions will usually be possible by a thorough history and examination.

Question 36 Tom is a 65 year old man who presents with fatigue and poorly localised muscular aches and pains in the back and legs. You suspect that he may be depressed but he denies feelings of depression. Which of the following alternatives would be the BEST way of confirming your preliminary diagnosis? a) Discuss Tom's symptoms with his wife b) Administer questionnaire

a

standardised

c) Undertake a therapeutic antidepressant medication

depression trial

of

d) Refer Tom to a psychiatrist e) All of the above

Answer: B A number of standardised questionnaires are available for the detection of depression. Among the simplest are those promoted by the Beyond Blue website, designed to assist GPs in the diagnosis of depression, viz the K10 and SPHERE questionnaires. The other options could all be helpful but are not recommended diagnostic strategies in themselves. Referral to a psychiatrist is only recommended for problematic or severe cases.

Question 37 Molly is a 34 year old woman who presents with chronic back pain following a fall at work one year ago. She is seeking a repeat prescription for oxycodone (Oxycontin), a powerful long-acting opiate analgesic. She is not receiving any other mode of treatment and is not undertaking back exercises. Molly is a trained nurse but has been unemployed since the accident because of disabling pain and is seeking a disability pension. What would be the most appropriate NEXT STEP toward solving Molly's problem? a) Provide a repeat prescription to reduce the number of times she needs to come to see you b) Help her complete the necessary paperwork for the pension c) Detail her drug use and assist her to switch to non-narcotic analgesia d) Refer her to a multi-disciplinary pain clinic e) Encourage her to begin back strengthening exercises

Answer: D Molly's problem is chronic because of its duration. Her case raises several issues: (i) How severe is her pain and does she really need a powerful analgesic? (ii) Is she misusing her prescription because she has become

dependent? (iii) What is the best approach to her problem? The best way of answering these questions is to refer her to a multi-disciplinary pain clinic, where she can obtain the benefit of assessment and advice by appropriate experts, as necessary, e.g. orthopaedic surgeon, anaesthetist (specialising in pain management), psychiatrist. The other options could assist but are unlikely to provide a solution to Molly's problems.

Question 38 Roberto is a 67 year old patient who is depressed following the recent death of his wife, and has moved to live with his daughter and son-in-law, because of difficulty in coping with living alone. You prescribe fluoxetine, a selective serotonin reuptake inhibitor (SSRI), but after 6 weeks of treatment, Roberto is still depressed. What is the MOST LIKELY reason for Roberto's failure to respond? a) A different SSRI would have been effective b) SSRIs are not the appropriate type of drug for this patient c) He has severe depression requiring specialist management d) He has psychotic depression electroconvulsive therapy (ECT)

requiring

e) His bereavement and loss of independence have not been dealt with

Answer: A The next step is to confirm or dispel your suspicions of domestic violence by directed but non-judgemental questions about the domestic situation, in particular how Cherie and her husband work out disagreements; whether she feels safe at home, and so on. It is preferable to question both parties if possible. Once a diagnosis of domestic violence is made, it is important to establish a supportive doctor-patient relationship and formulate a safety plan with the victim, including provision of information about abuse, the likelihood of recurrence, access to shelters and support groups etc. The option of informing the police and State authorities should be discussed and appropriate action taken according to the circumstances of the case. The prime consideration in domestic disputes is the safety of the victim and the children.

Question 40 Muriel is an 85 year old nursing home resident. The nursing staff are concerned about her, as she has vomited several times today and this afternoon complains of abdominal pain. She is not clear about its location, but it appears to be right-sided. She does not have a fever. Which of the following statements is CORRECT? a) It is important to have a high index of suspicion for gall bladder disease b) Muriel has early gastroenteritis c) Appendicitis is less common in elderly patients, but the risk of perforation is

Answer: E

also low

In patients like Roberto, psychological reactions to changed life conditions are likely to play an important part in the causation of depression. He has suffered bereavement and the loss of his independence, both of which are likely to be significant factors. Hence , psychological approaches to management (e.g. cognitive behavioural therapy) are more appropriate.

d) The most likely diagnosis is mesenteric ischaemia e) Muriel probably has diverticulitis, as 85% of cases involve the ascending colon

Answer: A Question 39 Cherie is a 38 year old married woman with two young children. She consults you because of anxiety which she attributes to 'the kids getting on my nerves.' You notice that she has a black eye and bruising of her left forearm, consistent with a defence injury. When asked for an explanation, she says she walked into the door of an open cupboard in the dark, when getting up to attend to the younger child at night. You suspect domestic violence. What is your next step in making a diagnosis? a) Non-judgemental domestic conditions

questioning

about

b) Report suspicions to the police c) Report suspicions to relevant State authority d) Provide information about shelters and support groups e) Offer family counselling

Elderly patients may present very differently from their younger counterparts and their abdominal pain is frequently misdiagnosed. However, approximately 3550% of patients older than 65 have gallstones, and may have associated biliary tract disease. The mortality rate for elderly patients with cholecystitis is approximately 10%, so a high index of suspicion for gall bladder disease is important, especially as symptoms and signs are often not classical. Although relatively common, a positive diagnosis of gastroenteritis should only be made after other potential causes have been considered and rejected.- Gastroenteritis in this age group should be a 'diagnosis of exclusion'. Appendicitis is less common in the elderly, with only 10% of cases being in the over 60 age group. However, the risk of perforation is approximately 50%. Mesenteric ischaemia is rare, but has a high mortality. Vomiting and diarrhoea are often present, but the pain in this condition is severe. Diverticular disease is common in the elderly, but diverticulitis- involving at least micro-perforation of the colon, - occurs in 85% of cases in left(descending) colon.

Block 6 Question 1

Answer: B

A 35 year old primigravida who's pregnancy had been uncomplicated, presents at 36 weeks with moderate oedema, BP155/95 mm Hg and ++ proteinuria. Which ONE of the following measures would you recommend?

Pityriasis rosea is a common mild acute inflammatory condition of the skin mainly affecting young adults. The cause is unknown, but a virus is suspected. The oval salmon-pink or copper-coloured eruptions with scaly margins are confined to the trunk and upper limbs and are often arranged along the skin creases (resulting an an appearance of a Christmas tree). Itching is usually mild. Lesions may be preceded by a 'herald patch' and disappear spontaneously in 4-10 weeks. For information on the other options see the references below.

a) Bed rest for the remainder of pregnancy b) Induction of labour without further delay c) Vigorous antihypertensive therapy d) Review again at 37 weeks e) Urgent referral to a specialist obstetrician

Answer: E This is a high risk patient on two counts, viz. her age at first pregnancy and the fact that she has pre-eclamptic toxaemia. These require specialist management in the latter stages of pregnancy and delivery.

Question 2 Which of the following conditions is FREQUENTLY associated with prerenal acute renal failure? a) Acute pyelonephritis b) Severe dehydration c) Rhabdomyolysis

Question 4 Bill is a 52 year old man who presents with nocturnal heartburn that has left him exhausted due to lack of sleep. Which of the following statements is CORRECT in relation to his gastroesophageal reflux disease (GORD)? a) Gastric acid hypersecretion is present in all such patients b) Disturbed oesophageal motility increases his risk of oesophagitis c) H2 receptor antagonists are the preferred medical management d) Maintaining an oesophageal pH of less than 3 is optimal e) Bile salts are as deleterious oesophageal lining as gastric acid

to

the

d) Prostatic hypertrophy e) Renal artery obstruction

Answer: B Prerenal acute renal failure is caused by underperfusion of the kidneys, e.g. due to dehydration, haemorrhage or shock. Acute pyelonephritis, rhabdomyolysis and renal artery obstruction are causes of intrinsic acute renal failure, i.e. the cause lies in the kidney. Prostatic hypertrophy is an example of a postrenal cause of acute renal failure, due to obstruction to the outflow of urine.

Question 3 This 22 year old girl presents with a rash on her trunk which has been present for three days (see figure). It is mildly itchy. What is the MOST LIKELY diagnosis? a) Lichen planus b) Pityriasis rosea c) Lichen simplex d) Atopic dermatitis e) None of the above

Answer: B The common denominator for virtually all episodes of gastroesophageal reflux is the loss of the normal gastroesophageal barrier to reflux. This is usually secondary to a transient or permanent loss of lower oesophageal sphincter resistance (eg. Gastric distension with air or food, increased intragastric or intraabdominal pressure, and delayed gastric emptying). Disturbed oesophageal motility allows prolonged exposure of the oesophageal lining to acidic fluids which is a major risk factor in the development of oesophagitis in GORD. Gastric acid hypersecretion may be present in some cases. The preferred medical management when significant symptoms are present is the use of a proton pump inhibitor which will help to maintain the oesophageal pH above 4. Bile salts reduce the resistance of the oesophageal lining, but are not as deleterious as gastric acid.

Question 5 Edith is a 75 year old woman who complains that her eyesight isn't as good as it used to be. While testing, you notice a lens opacity. Edith has well-controlled Type 2 diabetes and has a past history of polymyalgia rheumatica 20 years ago. She has osteoarthritis of the knees managed with intermittent NSAID therapy. Which

of the following is the MOST significant factor in the development of her cataract? a) Advancing age b) Diabetes mellitus c) Steroid therapy d) Radiation e) Trauma

Answer: A Although all the options increase the chance of cataracts, the most significant factor is advancing age. About 50% of 65-74 year olds have lens opacities increasing to 70% of those 75 years and over.

Question 6 Leigh is a 60 year old woman who has been hypertensive for 5 years. Her BP now is 160/115 mm Hg. Recently she has been getting increasingly short of breath. Clinical assessment confirms congestive cardiac failure. Which of the following drugs would be preferred for management? a) Propranolol

Answer: B Difficulty swallowing (or dysphagia) is a functional problem and a barium swallow is preferable to an endoscopy in this instance. Observations on the barium swallow may suggest oropharyngeal or cricopharyngeal dysfunction (including misdirection of barium into the trachea or nasopharynx), prominence of the cricopharyngeal muscle, a Zenker's diverticulum or a narrow pharyngeo-oesophageal segment. Disordered oesophageal motility or structural abnormalities such as small diverticula, webs, and minimal extrinsic impressions of the oesophagus may be recognised only with motion-recording techniques.

Question 8 Francesco is a 52 year old man with a history of rheumatic fever as a child. He presents with shortness of breath on exertion without orthopnoea or discomfort at rest. His echocardiogram demonstrates an isolated mitral stenosis of moderate severity. A typical finding on cardiac catheterisation would be: a) Normal left atrial and normal left ventricular diastolic pressures b) High left atrial and normal left ventricular diastolic pressure

b) Verapamil

c) High left atrial and high left ventricular diastolic pressures

c) Diltiazem

d) Right ventricular hypertrophy

d) Lisinopril

e) Left ventricular hypertrophy

e) Felodipine Answer: B Answer: D Linisopril is an angiotensin converting enzyme inhibitor (ACEI). This is the treatment of choice, as it lowers systemic vascular resistance and venous pressure and reduces the levels of circulating catecholamines, thus improving myocardial performance. It is important to observe for first-dose hypotension. Calcium channel blockers (e.g. verapamil, diltiazem) may have a detrimental effect on left ventricular function in patients with heart failure. Non-selective beta blockers such as propranolol are not well tolerated in heart failure. However third generation beta blockers such as carvedilol as well as beta-1 selective agents metoprolol and bisoprolol improve symptoms and exercise tolerance as well as lowering the risk of progression of heart failure and death. They are started once the ACEI dose is stable and fluid status is optimal. The beta blocker starts at a very low dose and is slowly titrated up.

Question 7 In which of the following situations would a barium swallow be preferable to an endoscopy as a FIRST LINE investigation? a) Patient complains of coughing after meals b) Patient complains of difficulty swallowing c) Patient with nocturnal symptoms only d) Patient with bloating after meals e) Patient has water-brash

Mitral stenosis of moderate severity indicates narrowing of the mitral valve orifice area from 5cm to 2cm squared. This results in an increased left atrial pressure and dilatation of the left atrium but a normal left ventricular pressure. Eventually pulmonary vascular pressures may rise with associated worsening of symptoms and then right ventricular hypertrophy may ensue.

Question 9 Jean, a 52 year old woman, presents with hot flushes related to menopause. She has noted a recent loss of libido. Which of the following statements concerning the effects of menopause on libido is INCORRECT? a) Sleep deprivation, secondary to hot flushes and night sweats, can lead to depression b) Vaginal dryness and painful intercourse due to oestrogen deficiency are common c) As the menopause approaches, erratic periods or menorrhagia may impact on sexual desire d) Oestrogen deficiency heightens sensitivity to touch stimuli, causing pain and discomfort e) Reduced muscle tone of the pelvic floor can affect orgasm

Answer: D

Answer : C

Clinical manifestations of oestrogen deficiency include:

Gastroesophageal reflux is caused by a transient or permanent loss of lower oesophageal sphincter tone. As a consequence, it more often occurs after meals. Controlling acid secretion at all times with a proton pump inhibitor is the medical treatment of choice. The other factors may play a role but are not the most important.

1. Variable length of menses - erratic, frequent or prolonged - which can impact on sexual desire 2. Hot flushes and night sweats, which can cause chronic sleep deprivation and potentially depression 3. Vaginal atrophy which can lead to vaginal dryness, dyspareunia and secondary vaginismus due to fear of intercourse 4. Urethral/bladder atrophy and loss of pelvic floor muscle tone, which can cause urethral incontinence. Loss of pelvic floor muscle tone can also reduce vaginal sensitivity and muscle contractions involved in orgasm 5. Touch aversion or reduced sensitivity to touch stimuli in genital tissues

Question 12 Which of the following describes an ependymoma? a) A benign skin lesion of the face b) A central nervous system tumour c) A cystic structure in the upper eyelid d) A patch of discoloured skin e) A visible defect in the iris

Question 10

Answer : B

Which of the following statements is CORRECT? In motor neurone disease there is:

Ependymomas are central nervous system tumours affecting the brain and spinal cord, derived from the single layer of cells (the ependyma) lining the ventricles and spinal canal.

a) A long exacerbations b) Sensory distribution

history loss

of

following

remissions a

and

dermatomal

c) Focal epilepsy d) Sparing of lower motor neurons e) Involvement of upper and lower motor neurons

Answer : E The commonest form of motor neurone disease is amyotrophic lateral sclerosis (ALS), a sporadic condition of unknown cause characterised by relentless progressive degeneration of upper and lower motor neurones in the spinal cord, the somatic motor nuclei of the cranial nerves and the motor cortex of the brain. The sensory system is not involved. Remission in this disease is unknown.

Question 13 A healthy six year old child without cyanosis or dyspnoea on exercise is examined for migration to Australia. His pulse is 84 per minute, B.P. 100/60, radial pulse and jugular venous pressure normal and there is no evidence of cardiomegaly. On auscultation in the 2nd left intercostal space the 1st and 2nd heart sounds are audible with fixed splitting of the 2nd heart sound and a midsystolic pulmonary ejection murmur is heard. The MOST likely diagnosis is: a) Pulmonary stenosis b) Atrial septal defect (ASD) c) Innocent pulmonary ejection murmur d) Ventricular septal defect (VSD) e) Patent ductus arteriosus (PDA)

Question 11

Answer : B

Joan is a 46 year old woman who has a long past history of bloating and water brash. She usually takes an antacid after her meals. In your advice to Joan you

In an asymptomatic patient an ASD is often diagnosed as a loud P2 with fixed splitting and an ejection murmur heard in the pulmonary area due to increased blood flow to the right heart. A VSD large enough to produce these signs would be symptomatic and usually would cause cardiomegaly. Innocent pulmonary ejection murmurs do not cause fixed splitting of P2, and a PDA causes a continuous murmur. In pulmonary stenosis P2 is often soft or inaudible and the JVP is usually elevated.

would explain that the MOST important factor in managing moderate and severe reflux is: a) Avoidance of alcohol with meals b) Avoidance of spicy food c) 24-hour control of acid secretion d) Limitation of the size of meals e) Not lying down too soon after meals

Question 14

Answer: B

Female androgen insufficiency syndrome includes all of the following EXCEPT:

A wet saline mount is 30-80% sensitive for Trichamonas vaginalis in symptomatic women. A drop of vaginal discharge is placed on a slide with 1-2 drops of normal saline and examined under high power (x 400). The oval or flagellated protozoan is seen among a large number of white blood cells and epithelial cells. Its erratic, twitching motility may be precipitated by warming the slide, thus distinguishing it from the similarly sized polymorphs. Culture is 95% sensitive especially in asymptomatic men and women, but not widely available and results take 7 to 10 days. Trichomonads are often reported on Pap smears from the cervix with a sensitivity of 60 to 70%. However a high false positive rate means further testing is necessary to confirm the diagnosis. Management involves treatment with metronidazole or tinidazole 2G stat orally. Contacts should also be treated and evaluated for other sexually transmitted infections.

a) Persistent fatigue b) Reduced body hair c) Decreased libido d) Blunted motivation e) Dysphoria

Answer : B Reduced body hair is not a feature of female androgen insufficiency. The syndrome includes:

1. Diminished sense of well being, dysphoric mood and/or blunted motivation 2. Persistent, unexplained fatigue

Question 17

3. Decreased libido, pleasure and sexual receptivity

Yee Leng, aged 40 years, is a new patient in your practice. She tells you she has been experiencing migraine headaches for a number of years. Which of the following features would NOT be consistent with migraine as a cause of her headaches?

4. Potential bone loss, decreased muscle strength, changes in cognition/memory

a) Numbness or tingling in one arm and hand

Question 15 The SINGLE MOST important diagnostic sign of developmental dysplasia of the hip in the neonatal period is? a) An unusually wide perineal region

c) Failure of the thighs to abduct readily d) Failure of the thighs to externally rotate e) A clunk on abduction of the flexed thigh

Answer: E On examination of the newborn a clunk or jerk noticed on abduction or adduction of the flexed thigh is suggestive of developmental dysplasia of the hip (previously called congenital dislocation of the hip). Ultrasound examination gives useful information as to the relationship of the femoral head to the acetabulum and the existence of any acetabular dysplasia during the first 3 months of life. If not diagnosed in the neonatal period the older child presents with an abnormal gait, a shortened leg on the affected side, asymmetrical abductor creases, restricted hip abduction (especially in flexion) and a positive trendelenberg's sign.

Question 16 trichomoniasis

is

c) Pain behind one eye radiating to the neck d) Bilateral pain e) Vertigo lasting about 30 minutes

b) Asymmetrical skin folds in the thighs

To optimise management, effectively identified by:

b) A headache which lasted 10 days

MOST

a) Cytological examination of a cervical smear b) Microscopic examination of a 'wet prep' c) Stained smear of vaginal discharge d) Culture of vaginal discharge e) Histological examination of a punch biopsy of vaginal mucosa

Answer: B Migraine attacks can last from hours (most common) to days, but never weeks. There are several different clinical patterns of migraine. Attacks can vary from intermittent headaches, indistinguishable from tension headaches, to discrete episodes that mimic thromboembolic cerebral ischaemia. Symptoms may include visual auras, nausea, vomiting, tingling, generalized headache, vertigo and transient hemiparesis.

Question 18 Betty, who is aged 29 years and nulliparous, comes to you for a routine Pap smear. The cervix looks healthy and there are no abnormalities on clinical examination. A week later she returns for the pathology result which is reported as: 'Abnormal cells are present, consistent with a diagnosis of carcinoma in situ CIN 3.' What is the implication of this result? a) Betty has malignancy of the cervix and requires radiotherapy followed by hysterectomy and clearance of lymph nodes b) Betty has a malignancy of the cervix and hysterectomy is required c) The smear should be repeated after Betty has applied clindamycin vaginal cream for 7 days d) Betty requires a cone biopsy e) Betty requires a colposcopy

Answer: E

Question 21

Betty needs referral to a gynaecologist for colposcopy and directed biopsy with definitive treatment if CIN 3 is confirmed.

Bill, aged 45 years, has been experiencing headaches on and off for 5 years. Over the past 2-3 weeks he feels that he has had an almost continuous headache. Which of the following features would make you suspect a tension-type headache?

Question 19

a) He is unable to continue with daily work

Mary aged 29 years, has suffered from migraines for three years. She is keen to reduce the frequency of attacks and asks about trigger factors. Which of the following advice would you NOT offer her?

b) The headache wakes him in the middle of the night c) It is very rarely bilateral

a) Use relaxation techniques to manage stress

d) It is usually bifrontal or bioccipital

b) Eliminate known dietary Chocolate, red wine, MSG

e) He suffers associated nausea

triggers

eg.

c) Maintain a regular sleep pattern d) Restrict time watching television to less than 2 hours a day e) Avoid contraceptives

oestrogen-containing

oral

Answer: D There is no known link between watching television and migraine, except that some programs may be stressful. The other options refer to recognised trigger factors.

Answer: D Tension headaches are often bifrontal or bioccipital. They are believed to be a result of tension within the scalp muscles. These headaches are always innocent and can be associated with tight band sensations, pressure behind the eyes, and throbbing and bursting sensations. Precipitating factors include worry, noise, concentrated visual effort, fumes or depression. There are no abnormal physical signs, other than tenderness and tension in the nuchal and scalp muscles.

Question 22 Question 20 Which of CORRECT?

following

statements

about

syphilis

is

George is a nineteen year old man who presents with dysuria. On examination there is a thick yellow urethral discharge. Which ONE of the following is CORRECT with regard to gonococcal infection?

a) Syphilis is only acquired through sexual contact

a) Urine PCR is now the gold standard for diagnosing gonococcal urethritis

b) The typical primary lesion of syphilis is a painful, hard ulcer on the genitalia

b) Spread to involve epididymis is common

c) Definitive diagnosis is by culture of fluid from the ulcer

c) The incubation period is usually 14 to 21 days

d) Signs of secondary syphilis mucocutaneous lesions and non lymphadenopathy

d) Ceftriaxone IM as a single dose is an effective treatment

include tender

e) Once treated there is no need for follow up

the

prostate

and

e) Pharyngeal gonorrhoea is diagnosed by finding Gram-negative diplococci on a smear

Answer: D

Answer: D

Secondary syphilis is marked by mucocutaneous lesions and non tender lymphadenopathy. Syphilis may be acquired by skin to skin nonsexual personal contact and transplacental infection. The primary lesion of syphilis is a painless, hard chancre on the external genitalia. The traditional, preferred testing method is dark-field microscopy. Specimens are taken from active lesions and examined directly. Serology should always be taken to diagnose syphilis. These tests are complex and require expert interpretation. Serology is also used to monitor the effectiveness of treatment in syphilis. Penicillin, given by injection, is a very effective treatment. Repeated blood tests are necessary for at least a year after treatment to monitor possible treatment failure.

Routine treatment for gonococcal infection is either with ceftriaxone 250 mg IM as a single dose or ciprofloxacin 500 mg orally as a single dose. The symptoms of gonococcal infection usually appear within 2 to 10 days of infection, but the incubation period is sometimes much longer, and some men never develop symptoms. In men, infection can spread to the prostate gland, epididymis, and the testes, although this is not common. Gram-stained smears are only sensitive and specific for urethral smears in symptomatic men. Culture is still the gold standard with >95% sensitivity and the ability to test for antibiotic sensitivities. Since the organism dries out quickly and is fairly fastidious, samples must reach the laboratory within 12 hours. PCR testing is validated for endocervical and first catch urine specimens only but fails to allow subsequent antibiotic sensitivity testing

which is significant in light of emerging quinolone resistance. It has the advantage of not requiring live organisms, being less invasive as a screening tool and enabling a check for chlamydia on the same sample. PCR tests are 90-100% sensitive and 98-100% specific.

Question 23 Belinda, aged 44 years, presents complaining of heavy, prolonged periods (menorrhagia) and severe period pain (dysmenorrhoea) that has gradually become worse during the past year. Her periods are still quite regular. Which of the following possible causes is UNLIKELY? a) Adenomyosis

Question 25 John is a 35 year old man who has had four recent episodes of intense retro-orbital pain. You suspect cluster headache. Which of the following features would be LEAST consistent with this diagnosis? a) He always vomits with the pain b) He has ipsilateral nasal stuffiness c) He notices a drooping of the eye on the same side as the headache d) Three of the episodes occurred within 2 days e) Episodes seem to last from 10 minutes to 2 hours

b) Endometriosis c) Uterine cancer d) Fibromyoma e) Ovarian failure

Answer: E Ovarian failure presents as irregularity and scarcity of menstruation, rather than menorrhagia and dysmenorrhoea. Adenomysosis, endometriosis, uterine cancer and fibromyoma are all possible causes of menorrhagia and secondary dysmenorrhoea. Other causes include uterine polyps, intra-uterine contraceptive devices, pelvic inflammatory disease and cervical stenosis.

Answer: A Cluster headaches are recurrent bouts of excruciating pain centred around one eye and lasting for minutes to hours. The affected side of the face and nostril feel congested. Commonly, there is a transient ipsilateral Horner's Syndrome. Vomiting may be associated, but is not always present.

Question 26 In the treatment of persistent tension-type headaches, in which mild analgesics and relaxation techniques are insufficient, which of the following medications would be MOST suitable? a) Propranolol 40mg at night

Question 24

b) Paroxetine 20mg in the morning

Martha is a 20 year old woman who presents with a vaginal discharge, bilateral painful vesicles and ulcers on her external genitalia. There is tender inguinal lymphadenopathy. She also has systemic symptoms, including headache, myalgia and a temperature of 38.5 degrees Celsius. Which of the following is CORRECT?

c) Amitriptyline 10-75mg at night d) Diazepam 2-5mg at night e) Carbamazepine 100-200mg twice a day

a) Antiviral agents eradicate the virus

Answer: C

b) Antiviral agents have no effect on recurrence

In patients in whom simpler treatments, such as paracetamol, and relaxation techniques, have failed, amitriptyline (10-75 mg) at night is indicated in the treatment of persistent tension headaches. The other medications listed are not indicated. Diazepam (short term use) may be effective in middle-aged men, but generally should be avoided because of risk of dependence.

c) Antiviral agents do not hasten healing d) Topical lignocaine is beneficial for the pain e) The primary aim is to keep the lesions clean and dry

Answer: E This scenario is typical of primary genital herpes. The primary aim is supportive treatment by keeping lesions as clean and dry as possible while spontaneous healing occurs. Analgesics by mouth are often useful, particularly at night time. No treatment is available to eradicate the virus, but antiviral agents reduce viral shedding from lesions, hasten healing and reduce the risk of recurrence while being administered. The recommended regimen is valaciclovir 500mg twice a day for 5 days. Topical lignocaine and zinc creams should not be used.

Question 27 Which of trismus?

the

following

describes

the

a) A grimace b) Tonic spasms of the jaw muscle c) A triad of symptoms d) Paralysis of the tongue e) A facial tic

meaning

of

Answer: B

Question 30

Trismus is the prolonged tonic spasm of the muscles of the jaw. It is an involuntary early sign of tetanus. Prolonged voluntary clenching of the jaw with teeth grinding is called bruxism, and is associated with emotional stress.

Which of the following statements about management of chronic pain is INCORRECT?

Question 28 James is a 32 year old man who presents two days after experiencing his first epileptic seizure. He was well until the seizure which occurred without warning. Since then he feels that his left leg has become weaker. James had meningitis as a child, and two years ago sustained a fractured skull in a motor vehicle accident. There is a family history of epilepsy (brother and an uncle have epilepsy). Which of the following groups of investigations is MOST likely to help in establishing the cause of James' epilepsy? a) Metabolic magnesium

screen

+

EEG

+

plasma

b) Metabolic screen + CT scan + plasma magnesium c) Metabolic screen + EEG + CT scan

the

a) Depression is often an aggravating factor b) Antidepressants are only indicated in clinical depression c) Psychological assessment is important to a good outcome d) A history of nerve damage may indicate neuropathic pain e) Opioid medication may be required for some patients

Answer: B Antidepressant medication may be helpful in patients with chronic pain even if they do not have depressive symptoms. These include patients with diabetic neuropathy, tension headache, rheumatoid arthritis and chronic low back pain.

Question 31

d) Metabolic screen + drug screen + EEG

In a child with chickenpox which of the following drugs is MOST LIKELY to cause Reye's syndrome?

e) Metabolic screen + drug screen + CT scan

a) Paracetamol Answer: C

b) Aspirin

A metabolic screen will detect biochemical causes, such as electrolyte abnormalities, acid base disturbances, hypocalcaemia etc. The EEG will help establish the diagnosis of epilepsy, classify the type and identify the particular epilepsy syndrome. A CT scan will identify any underlying structural abnormality, e.g. brain tumour. A drug screen is only necessary if there is reason to suspect drug abuse. Magnesium is one of the components of the metabolic screen.

c) Codeine

Question 29 Which of the following statements about brain tumours is CORRECT? a) Frontal lobe tumours present late b) Contralateral limb weakness or sensory loss are due to frontal tumours c) Disturbances of speech nondominant temporal tumours

are

due

to

d) Impaired insight is due to midbrain tumours e) Mood disturbances usually relate to tumours in the parietal lobe

d) Penicillin e) Prednisolone

Answer: B Reyes Syndrome involves acute encephalopathy and fatty infiltration of the liver following an acute viral infection, including influenza and varicella. Foreign chemicals, especially salicylates (including aspirin), and intrinsic metabolic defects have also been implicated. The use of salicylates (eg aspirin) during an acute viral illness such as chicken pox (varicella) increases the risk of Reyes syndrome by as much as 35-fold.

Question 32 Turner syndrome includes all of the following features EXCEPT: a) Ovarian agenesis b) Webbing of the neck c) A predominantly "X0" sex chromosome pattern

Answer: A Often frontal lobe tumours remain asymptomatic or undiagnosed until they are very large, possibly because local dysfunction of this brain region causes subtle and non-specific neurological or behavioural deficits.

d) The possession of 45 chromosomes e) A chromatin-positive buccal smear

Answer: E

Question 35

Patients with Turner's syndrome characteristically have a 45 XO chromosome pattern, and streak gonads (ovarian agenesis). Characteristic features include short stature, webbing of the neck, cubitus valgus, widely spaced nipples and puffy hands and feet. The range of intelligence is normal. A chromatin-positive buccal smear requires two X chromosomes. One X chromosome is randomly inactivated early in embryogenesis. Therefore, the buccal smear in Turner's syndrome does not have a Barr body (ie it is chromatin negative).

Mandy, a 4 year old girl, is due to accompany her parents on a flight to England in two months time. Her mother is worried about the effect of air travel on Mandy's ears. Which of the following will NOT increase the likelihood of ear pain during the flight? a) A recent cold b) Nasal congestion c) Hay fever d) Recent otitis media

Question 33 Sarah is an 8 week old girl who has persistent regurgitation. Which of the following features suggests the need for further investigation? a) Sarah is underweight for her age b) Sarah regurgitates after every meal c) Sarah has episodes of uncontrollable crying d) Sarah arches her back on occasion and stops feeding e) Sarah was born 2 weeks premature

e) Perforation of the ear drum

Answer: E Normal function of the middle ear and tympanic membrane is maintained by a patent eustachian tube which keeps the middle ear and external canal pressures equal. Blockage of the eustachian tube, e.g. with secretions as in the common cold, results in pressure imbalances between the middle and external ear. During air travel, the resulting barotrauma may cause pain, deafness, vertigo, and tinnitus. All the options listed, except (e), can cause blockage of the eustachian tube.

Answer: A

Question 36

Regurgitation after every meal suggests gastroesophageal reflux, but of itself is not a worrying feature. Underweight for age, however, suggests failure to thrive and needs investigation. Unsettled and irritable behaviour is very common in the first 6-12 weeks of life. In isolation it is not a concern.

Sue, a three year old girl, presents with shortness of breath and wheeze that have developed over the last two days. Examination reveals an afebrile, moderately tachypnoeic child with widespread scattered wheezes on auscultation. You decide to administer a bronchodilator (salbutamol). The recommended method of delivery of salbutamol for Sue is:

Question 34 Which of the following is suggested by a history of transient episodes of vertigo, slurred speech, diplopia, and paraesthesia in a man aged 65 years? a) Basilar artery insufficiency b) Anterior communicating artery aneurysm

a) Syrup b) Nebuliser c) Breath activated inhaler d) Metered dose inhaler with a spacer e) Metered dose inhaler with a spacer and face mask

c) Hypertensive encephalopathy d) Pseudobulbar palsy

Answer: E

e) Occlusion of the middle cerebral artery

For the treatment of acute asthma in a child 6 years of age, a large volume spacer may be substituted. Salbutamol administered via these routes has been shown to be equally effective to nebulised salbutamol. Six puffs of salbutamol via MDI and spacer is the equivalent of a 2.5 mg nebule, while 12 puffs equals a 5mg nebule.

Answer: A Episodes of vertigo, diplopia and paraesthesia indicate alteration of brainstem function caused by basilar artery insufficiency. Middle cerebral occlusions usually presents with contralateral hemiplegia and homonymous hemianopia. Slurred speech or aphasia may occur in both conditions. Anterior communicating artery aneurysms are likely to result in emotional lability. Hypertensive encephalopathy comprises headache, confusion, stupor or convulsions. Pseudobulbar palsy presents with dysarthria, dysphagia and emotional lability.

Question 37

Question 39

In acute iritis (anterior uveitis), the pupil is:

The MOST common cause of persistent cough in children is:

a) Eccentric, constricted and reacts briskly to light

a) Chronic post viral cough

b) Concentric, dilated and reacts briskly to light

b) Asthma

c) Eccentric, dilated and reacts sluggishly to light

c) Post-pertussis

d) Concentric, constricted and reacts sluggishly to light e) Concentric, dilated and reacts sluggishly to light

Answer: D Anterior uveitis typically presents with a unilateral painful red eye, blurred vision, photophobia and tearing. There is a perilimbal flush and pupillary miosis with a sluggish response and increased pain during accommodation to light. This is due to spasm of the ciliary body and iris muscles. Slit lamp examination will demonstrate keratic precipitates on the cornea. Urgent treatment is needed to prevent complications.

Question 38 Oliver, an 8 month old boy, presents with a history of anorexia and recurrent upper respiratory tract infections. Physical examination reveals pallor and splenomegaly. His blood picture shows a hypochromic, microcytic anaemia with numerous target cells and nucleated red cells. What is the MOST LIKELY diagnosis?

d) Gastro-oesophageal reflux e) Passive smoking

Answer: A The commonest cause of persistent cough (particularly with a prominent nocturnal component) in children is post viral. It can also be associated with postnasal drip. However, care must be taken to exclude asthma which is a fairly common cause of persistent cough in children. Other less common conditions to be considered include gastrointestinal reflux, passive smoking and post pertussis.

Question 40 Trevor is a 2.4 kg male infant with Apgars 9:10 after a normal vaginal delivery. He becomes jaundiced at 12 hours of age. Which of the following conditions would be the MOST LIKELY cause of the jaundice? a) Gram negative septicaemia b) Jaundice of prematurity c) Biliary atresia

a) Thalassaemia major

d) Physiological jaundice

b) Iron deficiency anaemia

e) Rh incompatibility

c) Glucose-6-phosphate deficiency

dehydrogenase

d) Sideroblastic anaemia e) Sickle cell anaemia

Answer: A Thalassaemia major (homozygous beta-thalassaemia) is a severe disease which presents during the first year of life with: failure to thrive, intermittent infection, severe hypochromic, microcytic anaemia and signs of extramedullary haemopoiesis (hepatosplenomegaly and bone expansion). It does not present at birth because the production of foetal haemoglobin is not affected.

Answer: E Jaundice appearing in the first 24 hours of life is most commonly due to haemolytic disease of the newborn due to incompatibility to Rh, ABO or one of the other rare antigens. Other causes of early jaundice include transplacental infections such as CMV, toxoplasmosis and rubella. Jaundice of prematurity, physiological jaundice and septicaemia present most commonly between days 2-5. Biliary atresia presents with jaundice after the first week of life.

Block 7 Question 1

Answer: E

Mrs B is 55 years old and presents because of painful mouth ulcers (see figure). You diagnose aphthous ulcers. She is otherwise well. Which of the following statements about aphthous ulcers is INCORRECT?

Intracerebral haemorrhages tend to be dramatic and accompanied by a severe headache. However, there really is no clinical way of reliably distinguishing between an intracerebral haemorrhage and a thromboembolic infarction, as both produce a sudden focal deficit.

a) The cause is rarely found b) There is often a family history c) The ulcers often begin in childhood

Question 4

d) They are often an indicator of Behcet's syndrome

Bel is 20 years old and has had a Type I allergic reaction to a bee sting. She states that there is a family history of bee sting allergy. Which of the following is NOT useful advice for Bel?

e) Aphthous ulcers may present like herpetic vesicles

a) Do not drink out of an open soft drink can that has been left outdoors Answer: D Behcet's syndrome is a systemic condition with recurrent multiple apthous ulcers AND two of the following: - recurrent genital ulceration, eye lesions, skin lesions or positive pathergy test (non-specific inflammatory skin reaction following intradermal saline injection). Recurrent apthous ulceration in isolation is not associated with Behcet's syndrome. The other statements concerning apthous ulcers are all true.

b) Have a supply of antihistamines on hand c) Insect repellents are useful to prevent bee stings d) Do not walk barefoot around swimming pools e) Always carry an adrenalin 1:1000 injection, e.g. EpiPen, and know how to use it

Question 2

Answer: C

The drug of choice for treatment of erysipelas is:

Insect repellents have not been shown to be useful in preventing bites from stinging insects. Anyone with a known allergy to stinging insects should know how to administer adrenalin 1:1000 subcutaneously and have it with them at all times. EPIpen is a commercial preparation which is supplied with an auto-injection device. Avoiding behaviours likely to lead to a sting such as those mentioned in the options and avoiding colourful clothes and perfumes which attract insects - is also important.

a) Penicillin b) Methicillin c) Ampicillin d) Tetracycline e) None of the above

Answer: A Erysipelas is a form of superficial cellulitis of the skin with lymphatic involvement. It is almost always caused by Strep pyogenes and therefore the treatment of choice is penicillin (erythromycin or cephalexin can be used in penicillin allergic patients).

Question 5 John is a 30 year old professional athlete who suddenly develops persistent dull upper left chest pain which is not related to exertion. There are no associated cardiac or respiratory features except for a mild restriction in breathing. John is afebrile and otherwise well. Which of the following diagnoses is LEAST likely?

Question 3

a) Spontaneous pneumothorax

Which of the following criteria enable a clear distinction to be made between haemorrhage and thrombosis in a patient with a cerebrovascular accident?

b) Functional chest pain

a) The progress of the clinical features b) The degree of loss of consciousness

c) Costo-chondral syndrome d) Muscle strain e) Pleurodynia ( Bornholm's disease)

c) The abruptness of onset d) The presence or absence of headache e) None of the above

Answer: E Bornholm's disease is due to an infection by Coxackie B virus. It is often associated with an acute upper

respiratory tract infection with fever, pleuritic chest pain and upper abdominal pain. These pains can be severe and associated with tachypnoea. A spontaneous pneumothorax, functional chest pain, costochondritis or acute muscular strain would be more likely in this patient.

Question 6 Which of the following statements concerning bee sting allergy is CORRECT? a) If a patient has had a life-threatening episode in the past, he or she is at risk of a future one b) A mild reaction in the past puts a patient at risk of a future life threatening episode c) Wasps only sting once d) A bee sting is smaller in volume than a wasp sting e) A raised serum IgG persists for years in those at risk of anaphylaxis

Answer: A A previous life-threatening reaction is an indicator that a similar episode may occur in the future. Reactions don't necessarily escalate. Wasps produce a smaller volume of sting, but are capable of stinging multiple times. Serum IgE levels remain elevated in those at risk of an anaphylactic reaction. IgG appears protective.

Question 7 The nephrotic syndrome can be caused by all of the following EXCEPT:

a) The causative organism is more likely to be bacterial than viral b) There is a higher risk of complications c) This may be the first presentation of inflammatory bowel disease d) Stool microscopy and culture to identify the organism is worthwhile e) All of the above

Answer: E The presence of blood in the stool of a patient with a constitutional illness is strongly suggestive of a bacterial infection. Identification of the causal bacteria is usually desirable for clinical and public health reasons. Most of the conditions are reportable to the relevant state health department. In some cases fulminant infection can occur and treatment based on antibiotic sensitivities of the organisms will be required. Many of these bacteria (Yersinia, Campylobacter, Shigella etc) can lead to a systemic illness with polyarthropathy.

Question 9 Abdul is a 58 year old man who presents with transient episodes of vertigo, slurred speech, diplopia, and paraesthesia. Which of the following is the MOST likely diagnosis? a) Basilar artery insufficiency b) Anterior communicating artery aneurysm c) Hypertensive encephalopathy d) Pseudobulbar palsy e) Occlusion of the middle cerebral artery

a) Minimal change disease (lipoid nephrosis) b) Amyloidosis

Answer: A

c) Membranous glomerulonephropathy

Transient ischaemic attacks involving the posterior brain circulation, i.e. the basilar artery, are characterised by diplopia, vertigo, vomiting, dysarthria, ataxia and hemisensory loss.

d) Renal vein thrombosis e) Diabetic nephropathy

Answer: D Renal vein thrombosis is more likely to be a complication of nephrotic syndrome rather than a cause. In nephrotic patients the blood is more coagulable than normal and the circulation may be sluggish owing to hypovolaemia, both of which are likely to induce thrombosis. The other options, together with focal and segmental glomerulosclerosis, are responsible for 90% of cases of nephrotic syndrome. They are all conditions which disturb the structure or function of the glomerular basement membrane.

Question 8 Bill, aged 35 years, has been unwell for 2 days with severe abdominal cramps and diarrhoea. He visited friends living on a farm during the previous week. He stated that on inspection of his stool there appeared to be blood in it. What is the significance of blood in the stool?

Question 10 Andrew, aged 60 years, complains of traces of blood in his stool for the past few weeks. Which of the following is the MOST likely cause? a) Diverticulosis b) Cancer of the rectum c) Haemorrhoids d) Angiodysplasia e) Colitis

Answer: C Haemorrhoids and/or anal fissures are the commonest and therefore most likely causes of traces of fresh blood in the stool. These are easily identified by physical examination. However, there are a number of other more serious pathologies which must be excluded,

including colorectal cancer diverticulosis, angiodysplasia and colitic conditions. These can be diagnosed by a combination of rectal examination, sigmoidoscopy and colonoscopy plus biopsy.

d) Pregnancy e) Chronic hepatitis

Answer: B Question 11 Cyanosis is LEAST likely to occur in: a) Ebstein's anomaly (anomalous attachment of tricuspid valve) b) Tetralogy of Fallot c) Tricuspid atresia

The ESR is a non-specific indicator of inflammatory and neoplastic disease. The ESR increases with age and is raised in pregnancy, the puerperium and in anaemia. It is increased in acute and chronic inflammatory disease and neoplastic disease. A low ESR (50%) and metronidazole can no longer be recommended for first-line therapy if there is no allergy to penicillins. The combination of a standard dose of PPI, amoxicillin 1G and clarithromycin 500mg all delivered bd for 7 days gives an eradication rate of >90%. It is available as a single script on the Pharmaceutical Benefits Scheme as either "Klacid Hp7" or "Nexium Hp7". In penicillin hypersensitivity the amoxicillin is replaced with metronidazole 400mg bd with efficacy 8085%. In treatment failure, there may be attempts to conduct bacterial culture and sensitivity testing. Otherwise a trial of quadruple therapy using a PPI, bismuth subcitrate, metronidazole and tetracycline for 10-14 days will give 75% chance of eradication. See references for further discussion about when to consider H pylori testing and eradication and how to follow up treated patients.

Question 35 A 67 year old man presented three days after a stent was inserted for the treatment of persistent angina. He now complains of a persisting "different" chest pain and shortness of breath on exertion. On examination you find he is pale and slightly sweaty with: pulse rate 110 regular with pulsus paradoxus; BP 100/90; T 38.0 degrees Celsius; pedal oedema; bilateral basal crepitations in his chest; and a JVP elevated 3cm. Heart

Question 36 A 60 year old engineer was admitted to hospital because of fever, cough, and pleuritic chest pain. His temperature was 40 degrees Celsius. Physical examination and x-ray of the chest indicated right lower lobar pneumonia. Sputum smear and culture demonstrated pneumococci. Treatment with intravenous penicillin was commenced. After several days, fever and leucocytosis decreased and x-ray of the chest showed some clearing of infiltrate. On the 7th hospital day, his temperature spiked to 39.4 degrees Celsius, there was an increase in cough and dyspnoea. X-ray of the chest showed an increase in pulmonary infiltrate. Which of the following is the MOST LIKELY explanation of this clinical picture? a) Development of pneumococcal resistance to penicillin b) Laboratory contamination of the original culture c) Superinfection by a different type of bacteria d) Adverse effect of antibiotic therapy e) Pulmonary thromboembolism

Answer: D This clinical picture demonstrates deterioration in the patient's condition. Drug fever, or serum sickness, usually occurs on the 7th to 12th day of antibiotic therapy and can produce unexpected fevers, skin rash and an eosinophilic pulmonary infiltrate. It is commonly due to penicillins, nitrofurantoin, sulphonamides, thiazides and tricyclic antidepressants. As he had been improving over 6 days, it is unlikely that the original specimens were contaminated, nor that the organisms had had the opportunity to develop resistance. Pulmonary embolism does not cause a high fever. Superinfection is commonly due to gram-negative bacteria, fungi or resistant staphylococci and usually appears on the 4th or 5th day.

Question 37

Question 39

Which of the following is NOT a likely complication from gastro-oesophageal reflux disease?

Brian and his wife are contemplating a trip across Africa. Which of the following situations is most likely to put them at risk of contracting typhoid fever?

a) Achalasia of the oesophagus

a) Intimate contact with an infected person

b) Ulcerative oesophagitis

b) Eating contaminated food

c) Barrett's oesophagus

c) Swimming in contaminated water

d) Adenocarcinoma of the oesophagus

d) Contact with an infected dog

e) Oesophageal stricture

e) Scratches from infected wildlife

Answer: A Achalasia is a motility disorder of the lower oesophagus of unknown cause and is unrelated to gastrooesophageal reflux disease (GORD). It is characterised by a dilated lower oesophagus in the presence of a tight lower oesophageal sphincter which does not respond to normal peristalsis. GORD leads to chronic inflammation of the lower oesophagus, which in turn can ulcerate or cause a fibrosed stricture. Barrett's oesophagus (or metaplasia) is also a result of long-standing GORD, occurring in 10% of patients with reflux. 10% of patients with Barrett's go on to dysplastic then malignant change (adenocarcinoma) of the oesophagus.

Answer: B Salmonella typhi is spread primarily through consumption of contaminated food or water. It has no known animal host other than humans. Intimate contact with infected persons is a relatively uncommon mode of transmission. Swimming in contaminated water would not lead to infection unless the water was ingested.

Question 40 Which of the following is NOT a feature of Parkinson's disease? a) Bradykinesia

Question 38 If a patient is said to have odynophagia you would assume that:

b) Rigidity c) Difficulty starting a movement

a) It is painful to swallow

d) Hypophonia

b) It is painful to speak

e) Intention tremor

c) The patient has trouble swallowing and hearing d) The patient has aberrant taste sensations e) There is oesophagus

incoordinate

action

of

the

Answer: A Odynophagia is pain during swallowing as opposed to a difficulty swallowing (dysphagia). It is usually a symptom of mucosal destruction.

Answer: E Parkinsonism is a syndrome consisting of tremor, rigidity and bradykinesis. The tremor is most marked at rest, coarser than cerebellar tremor and is seen as a pill rolling of thumb over fingers. Bradykinesis involves slowness in initiating and executing movement and speech. The gait is characterised by a shuffling forward with flexed trunk (called a festinant gait). Speech is hypophonic with a characteristic monotonous, stuttering dysarthria.

Block 9 Question 1 Jeremy has just been born via normal vaginal delivery. His Apgar scores are being documented. These are the observations made at 1 and 5 minutes of age: 1 minute: blue and pale; heart rate 100beats/minute; good cry; coughs with stimulation; active Which of the following BEST indicates Jeremy's Apgar scores at 1 and 5 minutes respectively? a) 4 and 9 b) 5 and 10 c) 3 and 9 d) 4 and 8 e) 5 and 9

Answer: A 1 minute: blue and pale (0); heart rate 100beats/minute (2); good cry (2); coughs with stimulation (2); active (2). Score of 9. Please see reference to review Apgar scores.

Question 2 Richard is 4 years old. He has just fallen over in the playground at kindergarten and knocked out (avulsed) his upper left front tooth. The tooth has been found on the ground. Richard's teeth are all primary teeth. He has not lost any teeth prior to this. Which if the following statements describes the BEST initial management? a) Immediately replace the tooth in its socket taking care to ensure its correct orientation b) Keep the tooth moist in saline, milk, or Richard's saliva and seek dentist review within the hour c) Carefully examine his mouth to check for other damage to teeth, bone or soft tissue d) Thoroughly clean the root of the tooth, wrap in plastic and transport on ice for dental review within 2 hours e) Prescribe antibiotics, preferably penicillin or clindamycin if penicillin-allergic

Answer: C Avulsed primary teeth are not usually reimplanted due to the risk of damage to the developing permanent tooth. Loss of a primary tooth rarely impacts on future occlusal development or tooth space. At least 50% of children will have a dental injury before 12 years of age. This may involve luxation of a tooth and soft tissue or alveolar bone damage or even mandibular fracture in

chin point trauma. It is essential that all teeth are checked and that any missing teeth or fragments are located. Radiographs may be necessary to check for ingestion, inhalation or soft tissue displacement of teeth. Avulsed permanent teeth should be kept in saline, milk or the child's saliva and replaced within half an hour. Antibiotics are used only if clinically indicated.

Question 3 Natalie is 12 years old. She presents with fever, an exudative pharyngitis, generalised lymphadenopathy and splenomegaly. This condition is MOST LIKELY caused by: a) CMV (cytomegalovirus) b) Group A streptococcus c) HSV (herpes simplex virus) d) RSV (respiratory syncitial virus) e) EBV (Epstein Barr virus)

Answer: E Natalie is most likely to have glandular fever (infectious mononucleosis) due to Epstein-Barr virus. Pharyngitis is the major symptom, and is exudative in 50% of cases. Splenomegaly is present in 50 - 75% of cases. CMV may cause a similar illness, but pharyngitis and adenopathy are less apparent. Pharyngitis due to Group A Streptococcus usually does not cause splenomegaly. HSV may cause gingivostomatitis but it rarely extends to the pharynx. RSV is a pathogen affecting the lower respiratory tract.

Question 4 Janet was born 3 hours ago at 35 weeks gestation. She has clinically apparent Down's syndrome. She has passed meconium normally. Janet has vomited several times, and the vomitus is bile-stained. Which of the following features would you expect to see on abdominal x-ray? a) Normal abdominal gas pattern b) Diffuse bowel distension c) Multiple dilated loops of bowel and intraabdominal calcifications d) Dilated proximal colon and absence of gas in the pelvic colon e) Dilated stomach and proximal duodenum with no air distally

Answer: E Janet has duodenal atresia, a condition commonly associated with other congenital abnormalities including Down's syndrome. Abdominal x-rays show the typical 'double bubble' appearance, due to distension of the

stomach and proximal duodenum with air, and the absence of gas distal to the obstruction. Diffuse bowel distension suggests meconium plug syndrome or Hirschsprung's disease (congenital megacolon). Hirschsprung's disease may also be seen as dilated proximal colon and absence of gas in the pelvic colon. Multiple dilated loops of bowel and intra-abdominal calcifications are seen with jejunoileal atresia or meconium ileus.

Answer: C The ductus arteriosus closes in normal term infants at 3 to 5 days of age. Patent ductus arteriosus is a common congenital cardiac abnormality, especially in premature babies. The incidence decreases with increasing maturity of the baby and is approximately 1:2000 in term infants.

Question 7 Question 5 Phillipa, aged 2 years, has just been diagnosed with autism. You are discussing the disorder with her distressed parents. Which of the following statements is CORRECT? a) Phillipa's parents have a 25% chance of having another child with autism b) Autistic children are extremely resistant to change in routines or patterns of behaviour c) The diagnosis of autism is increasing with a prevalence of approximately 2 per 10,000 children d) The range of intelligence of children with autism is the same as non-autistic children e) Autistic children often form a strong bond with their main carer and suffer marked separation anxiety

Answer: B Autism is one of a group of diagnoses called Pervasive Developmental Disorders. The incidence is as high as 26 per 1000 children. Parents of a child with autism have a 2-9% chance of having another affected child. Autistic disorder presents before the age of 3 years with sustained impairments in reciprocal social interactions, communication and restricted, stereotypical behaviour patterns. The lack of attachment or bonding to carers and resistance to change is typical. Up to 70% will have IQ's lower than normal. At the other end of the Autism Spectrum Disorders, children with Asperger's Syndrome may have normal or high IQ's, no language delay, are more adaptable but show impaired social interaction and restricted patterns of behaviour or interests. Early diagnosis and treatment has been shown to improve long-term outcomes for many children.

Question 6 Olivia is 7 days old. She was born at term in an uncomplicated normal vaginal delivery. She has a rough continuous 'machinery' murmur maximal at the left sternal edge, second intercostal space. Clinical findings are consistent with patent ductus arteriosus (PDA). When does the ductus arteriosus USUALLY close in a normal term infant? a) With the infant's first inspiration b) Within one hour of birth c) By day 3 to 5 of life d) By the age of six weeks e) Within the first six months

Benson is 4 years old. He was recently unwell with an influenza-like illness, from which he appears to have recovered completely. Benson has now developed marked periorbital swelling, and some peripheral oedema. He is not unwell, and has no other specific symptoms. Dipstick testing of Benson's urine reveals >3+ proteinuria. Which of the following is the MOST APPROPRIATE next step in the management of this condition? a) Renal biopsy b) Commence oral prednisolone c) Commence oral penicillin d) Commence oral diuretic therapy e) Reassurance and expectant management

Answer: B This presentation characterises a child with nephrotic syndrome. In young children, this is usually idiopathic nephrotic syndrome of childhood (minimal change disease). Renal biopsy is not usually performed unless there is poor response to treatment. The best initial management is corticosteroid treatment. Prednisolone is commenced at 2mg/kg/day (maximum 60mg/day) until urine protein levels fall to trace or negative levels, or for a maximum of 8 weeks, then therapy is tapered. Diuretics are rarely indicated and should be used with extreme care. Antibiotic prophylaxis may be considered during relapses, and pneumococcal vaccine may be worthwhile. Expectant management is inappropriate. Complications include infection, thromboembolism and hypovolaemia and all are treated aggressively. Some advocate prophylactic penicillin whenever there is severe oedema. Occasionally aspirin may be used to prevent clotting however the main strategy is mobilisation and avoidance of dehydration. About 70% of children have relapses but most of these will have ceased relapsing by 18 years of age. It is very rare for steroid-sensitive minimal change disease to progress to chronic renal failure.

Question 8 Catherine is a six week old baby. She presents to hospital with a history of vomiting after feeds, which her parents describe as projectile. Which of the following arterial blood gas (ABG) results is CONSISTENT WITH a diagnosis of pyloric stenosis? a) pH 7.41, pCO2 40, pO2 100, HCO3- 24 b) pH 7.51, pCO2 48, pO2 90, HCO3 30 c) pH 7.50, pCO2 29, pO2 105, HCO3 19 d) pH 7.19, pCO2 52, pO2 80, HCO3 18 e) pH 7.28, pCO2 60, pO2 85, HCO3 33

Answer: B

Answer: A

Infants with pyloric stenosis develop a hypochloraemic alkalosis with potassium depletion, secondary to vomiting. The compensatory respiratory response is hypoventilation to retain carbon dioxide. The parameters given in option (a) represent a normal ABG analysis. Option (c) represents a respiratory alkalosis with metabolic compensation, (d) shows a mixed metabolic and respiratory acidosis, and option (e) is an example of respiratory acidosis with metabolic compensation.

Genu varum (bow-legs) is common in toddlers to the age of 3 years. If asymmetrical or the intercondylar distance at the knees when standing is greater than 6 cm, Xray and referral is advisable to exclude the possibility of Blount's disease or rickets. Genu valgum is common between 3 and 7 years of age. The majority improve spontaneously. Refer if the intermalleolar distance at the ankles is greater than 8cm when standing. Femoral anteversion usually leads to toeing in due to excessive internal rotation of the femur. Corrective footwear is not necessary, nor is physiotherapy.

Question 9 Steven is 8 years of age. He is miserable and complaining of a sore throat. He has a fever of 39.2 degrees Celsius, tender cervical lymphadenopathy and tonsillar exudate with no cough. Steven has a documented Type I hypersensitivity reaction to penicillin. Which of the following statements describes the BEST management of Steven's condition?

Question 11

a) Regular paracetamol, fluids and expectant management

Timothy is 2 years old. He presents with a small crop of lesions on his left outer thigh, just above the knee. The 20 or so lesions are pale papules ranging in size from 2 to 5mm in diameter. Each papule has a central umbilication. They do not seem to be itchy or bothering Timothy, but his parents are very worried about them. Which if the following statements is TRUE?

b) Phenoxymethylpenicillin 250mg orally 12 hourly for 10 days

a) Timothy is being abused, as this is a sexually transmissable disease

c) Amoxycillin 375mg orally 8 hourly for 10 days

b) A moderately potent topical steroid is the treatment of choice

d) Roxithromycin 100mg orally 12 hourly for 10 days

c) Timothy should be investigated for causes of immunodeficiency

e) Cefaclor 250mg orally 8 hourly for 10 days

d) Resolution may take from 3 months to 5 years e) The most likely causative pathogen is human body louse

Answer: D Steven has tonsillitis and known penicillin allergy. On the basis that his symptoms are suggestive of streptococcal infection, Steven would benefit from antibiotics and the best management option is roxithromycin. Phenoxymethylpenicillin and amoxycillin are obviously contraindicated. Cefaclor is also contraindicated on the basis of possible cross-reactivity in a patient with a well-documented penicillin allergy. Regular paracetamol and fluids may well be of benefit, however antibiotics will improve symptoms and reduce the incidence of complications in bacterial tonsillitis.

Question 10 Robert is 4 years of age. He is developing normally and is of normal height and weight. However his mother is concerned as she has noted that he seems to be quite 'knock-kneed'. Robert's physical examination finds an intermalleolar distance of 6cm when he is standing with knees together. He is otherwise normal. Which of the following statements is CORRECT regarding genu valgum in this case? a) Reassurance is appropriate represents normal development

as

this

b) Robert should be immediately referred for an orthopaedic opinion c) Xray may confirm the cause as femoral anteversion d) Podiatrist advice regarding an appropriate orthotic should be sought e) Physiotherapy has been shown to improve prognosis if commenced before age 5

Answer: D Timothy has molluscum contagiosum. This is a poxvirus, causing characteristic lesions(firm, smooth, round, usually flesh-coloured papules with a central indentation or 'umbilication'). Chemical therapies are usually ineffective in treating these lesions, and conservative management is usually best. Resolution may take up to two years. The virus is spread by autoinnoculation, and severe or widespread disease may require more active management. This is a very common disorder in childhood. It may be spread by sexual contact in adolescents and adults, but is usually acquired from family members and other children in childhood. Molluscum contagiosum may be seen in advanced stages of HIV infection, but is not a flag to investigate for HIV in this age group. Question 12 Victor is a healthy 6 month old baby. He has a left hydrocoele, present since birth. It is relatively large and brilliantly transilluminable. Which of the following statements is CORRECT? a) The swelling is usually reducible b) The swelling is frequently painful c) Persistent swelling at 6 months requires surgical correction d) The swelling will usually resolve in the first years of life e) Aspiration of the swelling is usually curative

Answer: D

Answer: B

The natural tendency for the processus vaginalis to close in the first years of life will usually cause resolution of the hydrocoele. Aspiration may temporarily reduce the swelling but poses a risk for bleeding and secondary infection. Surgery is usually only required for a hydrocoele persisting beyond 2 years of age, unless it is extremely large. A hydrocoele is not reducible, and is not usually painful.

If foreign body aspiration is suspected, the patient should have a chest x-ray comprising inspiratory and forced expiratory views. Inspiratory views may show localised hyperinflation due to gas trapping distal to the foreign body. The forced expiratory film may show mediastinal shift away from the affected side. Bronchoscopy is indicated if a foreign body is detected, or if clinical suspicion persists despite negative chest xray.

Question 13 Nicholas is 14 years old. He has clinically apparent Marfan syndrome. Which of the following cardiac murmurs would you expect to hear on cardiac auscultation? a) Decrescendo high pitched diastolic murmur at left sternal edge b) Midsystolic ejection murmur at 2nd right intercostal space c) Low pitched rumbling diastolic murmur at the apex d) Pansystolic murmur at left sternal edge with no radiation e) Continuous machinery murmur at 2nd left intercostal space

Answer: A Marfan syndrome is an autosomal dominant connective tissue disorder with an incidence of about 1 in 10,000, of whom 25% present as a spontaneous mutation. Aortic dilatation causes aortic regurgitation, the murmur of which is usually decrescendo and high-pitched, beginning immediately after the second heart sound and extending for a variable time into diastole. A2 may be soft. Mitral valve prolapse may develop very early in life presenting as a midsystolic click. With time many go on to develop mitral regurgitation with an apical pansystolic murmur radiating into the axilla. The other options describe, in turn, aortic stenosis, mitral stenosis, ventricular septal defect and patent ductus arteriosis which are not features of Marfan syndrome.

Question 15 Stephanie is 12 months old. She has just begun to walk, but keeps falling over. It seems she has one leg slightly longer than the other. You suspect she may have congenital dislocation of the hip (CDH). Which of the following statements regarding this condition is TRUE? a) Ultrasound is the investigation of choice in a child of this age b) The incidence of CDH is approximately 1:100 live births c) After the first month of life, signs of hip instability become more evident d) Use of two or three layers of nappies (diapers) to splint the hip is frequently beneficial e) CDH is more common in female than in male infants

Answer: E Congenital dislocation of the hip is more common in female than in male infants. The incidence of CDH is approximately 1:1000 live births. After the first month of life, signs of hip instability become less evident, hence the condition is more easily missed after this time. X-ray is the best investigation for suspected CDH at the age of 12 months. Use of multiple nappies is never indicated as a treatment for this condition, as they are inadequate to obtain proper positioning of the hip.

Question 16 Question 14 Jake is 15 months old. His mother thinks he may have inhaled the wheel from a small toy car at a birthday party this afternoon. At the party he choked and coughed briefly then seemed to settle. However, since that time he has seemed a bit wheezy and has been coughing intermittently. Which of the following would be the BEST initial investigation to confirm the presence of a lower respiratory tract foreign body?

Trent is 14 years old and a keen athlete. He presents with left anterior knee pain. Tenderness of the left tibial tubercle raises your suspicion of Osgood-Schlatter disease. X-ray confirms your diagnosis and excludes other pathology. Trent's coach is keen for him to be treated immediately so he can resume training at full capacity. Which of the following represents the BEST management option for this condition? a) Reduction or modification of activity

a) Bronchoscopy

b) Electrotherapy

b) Chest x-ray

c) Corticosteroid injection

c) Arterial blood gases

d) Plaster cast immobilisation

d) CT scan

e) Surgery

e) White cell count

Answer: A

Question 19

Management of Osgood-Schlatter disease is conservative as this is a self-limiting condition. Ice and analgesics are appropriate for acute management of inflammation. The mainstay of treatment is to modify the child's level of activity to minimise aggravation of the condition. Complete abstinence from activity is unnecessary and undesirable.

Renee is 5 years old. She had a mild cold with a clear runny nose two weeks ago, which required no drug treatment, and has been well since. She presents with a purpuric rash on the fronts of her legs and feet, and her buttocks. The rash was urticarial initially. She appears well and happy, with no other abnormal clinical findings. Renee's platelet count is normal. What is the MOST LIKELY diagnosis?

Question 17 Christina is 4 years old. She has just been found to have drunk a good portion of a full bottle of children's paracetamol. Which of the following statements regarding paracetamol overdose in children is CORRECT?

a) Henoch Schonlein purpura b) Meningococcal septicaemia c) Idiopathic thrombocytopaenic purpura d) Disseminated intravascular coagulation e) Aplastic anaemia

a) The risk of hepatotoxicity is greatest in children under the age of 5 years b) Paracetamol levels are best measured as soon as possible post-ingestion c) N-acetylcysteine may be beneficial even when administered more than 24 hours postingestion d) Liver function tests are used to assess the need for treatment e) Normal liver function tests at 48 hours post ingestion are indicative of patient well being

Answer: C N-acetylcysteine is given to patients whose paracetamol levels are in the toxic range according to a standard nomogram, and may still be beneficial when administered more than 24 hours post-ingestion. The nomogram for calculating the requirement for Nacetylcysteine therapy is based on paracetamol blood levels, not on liver function test parameters. Significant abnormalities of liver function may not develop until 72 hours post-ingestion. The incidence of hepatotoxicity is ten times higher in adults and adolescents than in children under 5 years of age. If paracetamol is ingested in a liquid form, blood levels may be taken two hours after ingestion to give an accurate toxicity picture, otherwise the level should be taken at 4 hours postingestion.

Answer: A Henoch Schonlein purpura is the most common small vessel vasculitis in children. It is commonly preceded by a viral upper respiratory tract infection. Less commonly a drug may be implicated in its onset. The rash may be urticarial initially, indurating and becoming purpuric. The extensor surfaces of the feet and legs, arms and the buttocks are frequently involved. New lesions can appear for 2-4 weeks. This disease can also be associated with fever, polyarthralgia, abdominal pain and melaena, proteinuria and haematuria. The platelet count is usually normal, but may even be elevated.

Question 20 Harvey aged 10 years, was playing football this afternoon when he was heavily tackled. He had to leave the field as he was injured in the incident, and has had severe left flank pain since. He also complains of some dizziness. Examination confirms left flank tenderness. You suspect Harvey may have a ruptured spleen. Which of the following plain abdominal x-ray findings would CONFIRM this diagnosis? a) Loss of the left psoas shadow b) Upward displacement hemidiaphragm

of

the

left

c) Enlargement of the spleen d) Fracture of the left lower ribs

Question 18 Jordan is 5 years old and has cerebral palsy. He has a spastic hemiparesis. Which of the following features is NOT seen with this type of abnormality? a) Hypertonia b) Fasciculations c) Hyperreflexia

e) Absence of gas in the splenic flexure

Answer: A The left psoas shadow may be obscured by haemorrhage from a ruptured spleen. Lower rib fractures provide evidence of local trauma but are not diagnostic of splenic rupture.

d) Ankle clonus e) Extensor plantar reflex Answer: B In spastic cerebral palsy, the motor features are of an upper motor neuron abnormality. Fasciculations are seen with lower motor neuron dysfunction. The other options are correct.

Question 21 Jenny and Sam are beside themselves with worry. Their son, Justin, is 11 years old and in grade 6 at school. They are finding it increasingly difficult to encourage Justin to go to school. He frequently complains of being unwell in the mornings, with recurrent stomachache and headache. His symptoms tend to be less apparent at weekends and during school holidays, and often improve as the day goes on when he is allowed to stay at home.

This pattern of behaviour has been typical for Justin for some years, and he has been repeatedly examined and investigated. No organic cause for his symptoms has been diagnosed. He denies hallucinations. He performs well at school, and his teacher says he seems to get along well with his peers, with no evidence of bullying. Which of the following is the MOST likely cause of Justin's school refusal?

Answer: A Greg is likely to have an acquired autoimmune haemolytic anaemia, with the production of cold agglutinins in response to the EBV. This causes a normochromic normocytic anaemia, as well as the other features listed. Hypochromic microcytic anaemia is associated with iron deficiency.

a) Antisocial personality disorder b) Gender identity disorder

Question 24

c) Separation anxiety disorder

You are discussing the genetic inheritance of a condition with a family. Peter and Simone have just learned that their son, Paul, has Duchenne muscular dystrophy. Which statement is TRUE regarding the mode of inheritance of this condition?

d) Early onset schizophrenia e) Panic disorder

a) Autosomal dominant Answer: C

b) X-linked recessive

Justin's presentation most likely represents developmentally inappropriate separation anxiety. The absence of a realistic cause for his fear of school and the tendency for his symptoms to abate when allowed to remain at home, as well as the absence of any organic disease or positive psychotic symptoms (such as voices) support this.

c) Autosomal recessive d) Chromosomal deletion e) Chromosomal translocation

Answer: B Question 22 Alexander is 19 months old. He has not received any childhood immunisations as his parents are conscientious objectors to vaccination. Alexander has been unwell for several hours with fever, irritability and dysphagia. He presents drooling and leaning forward and is obviously distressed. What is the MOST LIKELY organism to be causing Alexander's condition?

Duchenne muscular dystrophy is a sex-linked disorder, inherited via an abnormal recessive gene on the X chromosome. Thus, only males are affected. Unaffected female carriers transmit the disease. For children of a carrier female, there is a 50% chance that each daughter will be a carrier, and a 50% chance that each son will have the disease. Affected males cannot transmit the disease to their sons, but all of their daughters are carriers.

a) Streptococcus pneumoniae b) Haemophilus influenzae type B

Question 25

c) Non-typable Haemophilus influenzae

Ngaire is a six week old baby girl. Her parents have noted a swelling in the right side of her neck. She seems well, is breastfed and is gaining weight. Which of the following should NOT be considered in the differential diagnosis of a lateral cervical swelling?

d) Streptococcus pyogenes e) Staphylococcus aureus

a) Acute cervical adenitis Answer: B

b) Branchial cleft cyst

Alexander has classic epiglottitis. All of the listed organisms are known to cause epiglottitis. However, Haemophilus influenzae type B is almost always the causative organism in an unimmunised patient.

c) Cystic hygroma

Question 23 Greg is 7 years old. He has recently had confirmed glandular fever (Epstein Barr virus). Over the past 3 days he has developed weakness, pallor, dark urine and fatigue. On examination, he is jaundiced and has mild splenomegaly. Which of the following is NOT a feature you would expect to find on laboratory investigation?

d) Sternocleidomastoid muscle haematoma e) Thyroglossal duct cyst

Answer: E Thyroglossal duct cyst should not be considered in the differential diagnosis of a lateral cervical swelling. This congenital abnormality occurs in the midline, between the hyoid bone and the suprasternal notch. The other options may present as a lateral cervical swelling.

a) Hypochromic microcytic anaemia b) Spherocytes

Question 26

c) Nucleated red blood cells

Cindy is 3 years old. She celebrated her birthday 3 days ago, and has been wearing her favourite present, a new necklace. Cindy has now developed a rash on her neck and anterior chest where the necklace rests against her skin. The rash is slightly raised, erythematous and itchy.

d) Hyperbilirubinaemia e) Elevated LDH

Which of the following statements best describes the MOST LIKELY explanation for this? a) Type I hypersensitivity b) Type II hypersensitivity c) Type III hypersensitivity d) Type IV hypersensitivity e) The rash is unlikely to be related to the necklace

Answer: D The rash is most probably a contact dermatitis brought on by the presence of the necklace. The most likely cause of Cindy's rash is a Type IV hypersensitivity reaction. This is a T-cell mediated immune response, and is usually a delayed response to a specific antigen.

Question 27 Rebecca's mother contracted rubella whilst she was pregnant, which was confirmed by serological testing. As a result Rebecca has been born with congenital rubella syndrome. Which of the following is NOT a feature of congenital rubella syndrome?

Answer: C In the previously healthy individual, thrombocytosis follows splenectomy. Anticoagulant treatment is required if the platelet count exceeds 10,000x 10*9/L to prevent thrombotic complications. All of the other statements are true. Pulmonary dysfunction may ensue because of trauma to ribs, the sub-diaphragmatic area and the lungs, from the accident. Post-splenectomy there is a greater susceptibility to overwhelming bacteraemia because of decreased bacterial clearance, levels of IgM and opsonisation of encapsulated bacteria. Pneumovax should be given pre-operatively if possible and immunisation to Haemophilus influenzae type B and Meningococcus should be updated. The pancreas may be damaged at the time of surgery, which could result in a pancreatic cyst or fistula. This may become infected giving rise to the subphrenic abscess.

Question 29 Stefan is 7 years old. He has been diagnosed with Attention Deficit Hyperactivity Disorder (ADHD). After careful consideration and evaluation, it has been decided to commence him on pharmacological therapy. Which of the following drugs is NOT used in the treatment of ADHD? a) Dexamphetamine b) Methylphenidate c) Imipramine

a) Growth retardation

d) Clonidine

b) Deafness

e) Diazepam

c) Limb hypoplasia d) Cardiac defects

Answer: E

e) Mental retardation

Answer: C Congenital infection with rubella causes a number of manifestations, the main ones being: mental retardation, cardiac anomalies, growth retardation, ocular anomalies, deafness, cerebral disorders including chronic encephalitis, and haematologic disorders. Limb hypoplasia is not usually seen. This may be a feature of congenital varicella zoster infection.

Question 28 Eloise, aged 16, has been involved in a serious motor vehicle accident in which she has suffered a ruptured spleen. Unfortunately her clinical signs deteriorate, and splenectomy is indicated. In obtaining informed consent from Eloise and her parents, you discuss possible postoperative complications. Of the following statements, each is true EXCEPT a) There is dysfunction

a

possibility

of

pulmonary

Question 30 Sally is 6 years old. She is being admitted to hospital for tonsillectomy and adenoidectomy. Informed consent is being sought from Sally's parents, and the risk of haemorrhage and the possibility of a blood transfusion being required are discussed. Sally's parents wish to know the risk of Sally contracting hepatitis C via blood transfusion. The risk of transmission of hepatitis C via blood transfusion is: a) 1:100 b) 1:1,000 c) 1:10,000

b) Pancreatic damage may occur at operation c) Thrombocytopoenia may requiring platelet transfusion

Diazepam is not indicated in the treatment of Attention Deficit Hyperactivity Disorder (ADHD). Dexamphetamine and methylphenidate are psychostimulants and may be considered as first line therapy. Imipramine is a tricyclic antidepressant which may be considered as second-line therapy only if psychostimulants are not tolerated, are contraindicated or are ineffective. Clonidine is widely used, although its effectiveness remains unproven.

follow

surgery,

d) 1:100,000 e) 1:1,000,000

d) Post splenectomy bacterial infection may be life-threatening

Answer: E

e) A subphrenic abscess may be a later complication

All donor blood products are screened for hepatitis C antibody. The estimated risks for false-negative tests for

hepatitis C on donor blood, and thus the risk of hepatitis C are approximately 1:1,000,000. The Australian Red Cross Blood Service tests all donations for HCV antibody as well as nucleic acid PCR so the current estimate of risk is 1:3,663,000.

Question 31 Paulette, a curious toddler aged 18 months is playing happily with Grandma's sewing box. As her mother approaches, Paulette swallows something she has had in her mouth. Grandma keeps a variety of objects in this box. Which of the following objects is MOST likely to require active intervention if Paulette has swallowed it? a) A two dollar coin b) A duck shaped button c) A hearing aid disc battery

Answer: B Daniel almost certainly has an intussusception, which is most common in children 3 months to 3 years of age and typically presents with the features described above. Air enema is an appropriate investigation and in ileocolic intussusception, often resolves the abnormality with less than 10% chance of recurrence. If unsuccessful, urgent surgery is necessary. In infants presenting with abdominal pain the differential diagnosis will include colic, gastroenteritis, constipation, urinary tract infection, volvulus and incarcerated hernia. The suspicion of a surgical condition increases if there is severe or increasing pain, bile-stained vomitus, guarding or abdominal rigidity and distension. The passage of blood and mucus producing th so-called "red-currant jelly" is often a late feature of ischaemic bowel in either intussusception or volvulus. Appendicitis is rare in infants but in young children there is up to an 80% risk of perforation at the time of diagnosis

d) A safety pin (probably closed) e) A small marking pencil 3 cm long

Answer: C The corrosive nature of the battery can lead to gastrointestinal erosion and/or perforation. This is a particular risk if the battery becomes lodged in the oesophagus as erosion can begin within six hours of ingestion. The majority of disc batteries, however, do pass uneventfully through the remainder of gut if they pass the level of the lower oesophageal sphincter. Coins and round buttons generally pass through the gut uneventfully .in a child., as do long items less than 6 cm There is a very small risk of perforation with sharp objects such as irregularly shaped buttons, or small toys, but the majority of these do not require active intervention. Even open safety pins will usually pass through the gut without harm.

Question 32 Daniel, a previously well 10 month old infant, has been severely distressed and screaming intermittently for the last 1-2 hours. The screaming bouts last for 2-3 minutes, and recur every 10- 15 minutes although his Mum says they are becoming more frequent. He draws his legs up when he screams appearing to be in pain, and is very flat and lethargic when he is not screaming. He has vomited three times in the last hour, the last vomit being a greenish fluid. What is the MOST appropriate advice to give his worried mother? a) Reassurance that colic is a common condition which may respond to simple treatments and H2 receptor antagonists b) An air enema is likely to be both diagnostic and effective as a treatment in this case c) Admission will be necessary to provide supportive treatment of this severe form of gastroenteritis d) Treatment includes analgesia and IV fluids but avoidance of invasive procedures unless there is per-rectal blood and mucus e) Urgent surgery is necessary due to the higher risk of appendiceal rupture in this age group

Question 33 Tasneem has just been born with symptomatic congenital cytomegalovirus infection (CMV). Which of the following is the MOST COMMON finding in this condition?? a) Hepatosplenomegaly b) Microcephaly c) Deafness d) Chorioretinitis e) Cataracts

Answer: A CMV is the world's leading cause of congenital virus infection. Approximately 1% of all infants are congenitally infected and CMV infection is symptomatic in 5-10% of these infants. Of symptomatic neonates, 60-80% will have hepatosplenomegaly, petechiae and jaundice; 30-50% microcephaly, intra-uterine growth retardation or prematurity; and the mortality is 20-30%. Survivors have a 90% chance of going on to develop sensorineural deafness, mental retardation, seizures and motor delay. Asymptomatic neonates may later develop microcephaly and psychomotor retardation with up to 25% becoming deaf. Chorioretinitis occurs in 20% of symptomatic neonates but cataracts are associated with congenital rubella infection, not CMV. Since 50-60% of women of childbearing age are carriers of CMV, then 40-50% of women are at risk of primary infection in pregnancy which may be transmitted transplacentally in up to 40%. For carriers, the risk of reactivation in pregnancy is estimated to be anywhere from one to 25% but risk of transmission to the foetus in this case is only 2-5% and is nearly always asymptomatic in the neonate

Question 34 Jennifer, age 25 years is concerned as she has been trying to fall pregnant for the last three months and has been unsuccessful. In giving her advice, which of the following statements is CORRECT? a) Pregnancy occurs in 80% of couples after 12 cycles; in those who have not conceived the chance is 50% in each year thereafter

b) Healthy young couples have a 10% chance of achieving pregnancy in one menstrual cycle, that is, a fecundability of 0.10 c) If a couple less than 30 years of age have not conceived in 12 months, they have no chance of conception d) With a regular cycle most women of this age have conceived by 3 months so gynaecological review is recommended e) Preliminary investigations to confirm ovulation and adequate sperm should now be undertaken by her general practitioner

Answer: A Infertility is defined as the failure of conception in a couple having regular, unprotected sexual intercourse for one year, provided that it is occurring not less than twice weekly, and menstrual cycles are regular.80% of couples will be pregnant after 12 cycles. Of those who have not conceived after 12 cycles, about 50% will conceive during a second year of attempted conception. After this second year of attempted conception, the chance of conception in those couples remaining is about 50% in the following four years. The fecundability of a young couple is approximately 0.25, that is, there is a 20-25% chance of pregnancy for each menstrual cycle. Fecundability for women declines from about 35 years of age, precipitously after age 40 Young women of this age with this presenting complaint should have a thorough history taken to exclude obvious causes of infertility and to reassure the woman that she is normal. It is important to clarify that coitus is achieving adequate penetration to optimise sperm delivery, and that timing matches the peak fertile period during her menstrual cycle. Pre-pregnancy counselling will include folate supplementation, rubella immunity and Pap smear testing. Investigation of the couple is usually deferred until 12 months have passed unless the woman is of advanced age.

Question 35 Monique, age 18 years, keeps forgetting to take her combined oral contraceptive pill and is keen to try depot medroxyprogesterone, as her friend is using it with great success. All of the following are side effects of this medication EXCEPT:

effects. There is some debate as to the significance of the slight fall in bone density seen in some studies of women using this form of contraception. Thrombosis is a side effect of oestrogens rather than progestogens. Question 36 Malka, age 16 years, presents with primary dysmenorrhoea which is preventing her from attending school when she has her period. With regards to the management of primary dysmenorrhoea which of the following is TRUE? a) Paracetomol is as effective as non-steroidal anti inflammatory drugs b) Combined oral contraceptives should not be offered unless the young woman is sexually active c) Non steroidal anti inflammatory drugs should be recommended as first line agents d) Depo provera is of no assistance in the management of primary dysmenorrhoea e) Combined oral contraceptives should not be used in conjunction with non-steroidal anti inflammatories

Answer: C Primary dysmenorrhoea results from uterine vasoconstriction, anoxia, and contractions mediated by prostaglandins. Non steroidal anti inflammatory drugs (NSAIDS- eg: Naprosyn (mefanemic acid) are recommended as first line agents in the management of dysmenorrhoea as they inhibit prostaglandin synthesis and therefore provide symptomatic relief. Paracetomol is not as effective as NSAIDS in this condition due to it's lack of effect on prostaglandin pathways. While it provides simple analgesia it does not address the causation of the problem. The combined oral contraceptive pill reduces menstrual flow and inhibits ovulation and is also effective in the treatment of dysmenorrhoea. It can be used with NSAIDS, although this is rarely necessary.. The lack of response to NSAIDs and OCs (or the combination) may increase the likelihood of a secondary cause for dysmenorrhoea. Depo provera may also prove helpful in the management of dysmenorrhoea as it inhibits ovulation and often results in amenorrhoea.

a) Weight gain b) Depression c) Thrombosis d) Amenorrhoea e) Delayed fertility

Answer: C Depot medroxyprogesterone acetate, marketed as Depo-Provera and Depo-Ralovera, is an injectable progestogen given every 12 weeks +/- 2 weeks. Women can suffer from progestogenic side effects such as weight gain, bloating, and menstrual disturbance. Initially women may suffer from irregular bleeding, however amenorrhoea occurs in 50% of women after 1 year of injections. There is usually a delay in the return of fertility until approximately 9 months after the last dose. Mood changes and acne are less common side-

Question 37 Maria, G3P3, 35 years is 4 weeks postpartum and breastfeeding. She seeks advice regarding postpartum contraception. Which of the following isTRUE? a) Maria should not recommence sexual activity until at least six weeks postpartum b) Lactational amenorrhoea is 98% effective for the first 6 months if fully breastfeeding (no solids) and amenorrhoeic c) Any of the oral contraceptive preparations may be safely used after 6 weeks post-partum d) Intra-uterine devices (IUDs) should not be used until the uterus is back to normal at 6 months post-partum e) Implantable progesterone-only contraception (Implanon) is contra-indicated in breastfeeding women

Answer: B Sexual activity can recommence whenever the woman feels comfortable enough to try it, taking into consideration the time needed for episiotomies and vaginal tears to heal. In terms of contraception, as long as the woman is fully breastfeeding (no solids) and has not started menstruating then lactational amenorrhoea provides 98% protection against pregnancy. Contraception containing estrogen such as the COCP is not recommended as it may reduce the volume of breast milk. Progestogen only methods of contraceptives are usually recommended as they are safe when breastfeeding. These include the "minipill", depot injections, Implanon or the Mirena IUD, all of which may begin from 6 weeks post-partum. IUDs are safe to insert 6 weeks after a vaginal birth when the uterus has returned to normal size and bleeding associated with the birth has stopped. IUDs should be inserted at least 12 weeks after a caesarean birth as there is a slightly greater risk of perforation of the uterus before this.

Question 38 Sally has come to discuss the combined oral contraceptive pill (COCP) with you. Which of the following is TRUE regarding some of the noncontraceptive effects of the COCP? It: a) Increases the risk of benign ovarian cysts b) Reduces the risk of endometrial cancer c) Has no effect on the risk of colorectal cancer d) Increases the risk of pelvic inflammatory disease e) Reduces the risk of invasive breast cancer

Answer: B The COCP has little effect on benign ovarian cysts. If anything, they may reduce in size. Ovarian cancer: The longer the use of the pill, the greater the protection. After more than 10 years on the pill, the risk of ovarian cancer may be reduced by 60- 80 %. This protective effect appears to last at least 15 years after stopping the pill. Researchers theorize that the pill may lessen ovarian cancer risk in part by inhibiting ovulation. Endometrial (uterine) cancer: In women using the pill the risk may be reduced by as much as 50%. This protection also lasts at least 15 years after the pill is stopped. Colorectal cancer: On the pill the risk may be reduced by approximately 35%. Researchers believe the protective effect may be due to a reduction in the concentration of bile acids in the colon. Latest research suggests that the COCP has no effect on the risk of breast cancer. Pelvic inflammatory disease (PID): Although rarely fatal, this infection can lead to infertility from scarring of the fallopian tubes. The pill cuts the risk of PID by half. It is believed the pill works by making cervical mucus an unfriendly environment for disease-causing bacteria.

Question 39 Mandy has requested you insert a copper-bearing IUD. She is 34 years of age, with regular 28 day menstrual cycles and she is in a stable monogamous relationship. Mandy wants to know when, during her next menstrual cycle she should have the IUD inserted. Which of the

following is the CORRECT advice to regarding timing of insertion of an IUD?

give

Mandy

a) An IUD can be inserted any time during the last 14 days of her menstrual cycle and no additional contraceptive protection is needed b) An IUD should only be inserted whilst Mandy has menstrual bleeding and no additional contraceptive protection is needed c) An IUD can be inserted any time within the first 12 days after the start of menstrual bleeding and no additional contraceptive protection is needed d) An IUD can be inserted at any stage within the middle two weeks (14 days) of her menstrual cycle and no additional contraceptive protection is needed e) An IUD can be inserted any time during her menstrual cycle provided she uses 7 days of additional contraceptive protection after insertion

Answer: C A copper-bearing IUD can be inserted any time within the first 12 days (Day 1 = the first day of menstrual loss) after the start of menstrual bleeding. No additional contraceptive protection is needed. The probability of an existing pregnancy is extremely low before day 12 of the menstrual cycle based on the extremely low risk of ovulation before day 8, plus the copper-bearing IUDs provide 5-days of emergency contraceptive effect. These recommendations do not apply to hormonal IUDs as their emergency contraceptive effects if any, are unknown.

Question 40 Beth wants to start using DMPA as a form of contraceptive. Which of the following statements regarding Depot Medroxyprogesterone Acetate (DepoProvera, Depo-Ralovera, depot MPA, DMPA) is TRUE? a) DMPA is given as a subcutaneous injection b) DMPA always makes women amenorrhoeic c) DMPA causes a delay in return of fertility d) Prolonged DMPA use increases the risk of endometriosis e) Prolonged DMPA use increases the risk of ovarian cancer

Answer: C Depot Medroxyprogesterone Acetate (Depo-Provera or depot MPA or DMPA) is a long-acting hormonal contraceptive which is usually given as an intramuscular injection once every three months. Disadvantages of DMPA include changes in the menstrual cycle during treatment, usually resulting in oligomenorrhoea or amenorrhoea, but sometimes resulting in troublesome, irregular bleeding, especially in the first few months of use. DMPA can also cause an unpredictable but temporary delay in return of fertility following treatment. There is increasing evidence for substantial protection against endometrial cancer, ovarian cancer,

acute pelvic inflammatory disease and endometriosis. There is no evidence for serious long-term complications of treatment with DMPA, although there is current controversy about the possible small risk of increased

bone loss in women using DMPA long-term, who have continuous amenorrhoea and low serum oestradiol levels.

Block 10 Question 1 Prolonged neurological sequelae in infants can occur if the mother has taken which of the following in the third trimester?

orbicularis oculi). In isolation, the dystonias are usually of unknown cause and treatment is difficult. Dystonias as a reaction to medication are much more common in children, for example, metoclopramide (an anti-emetic) results in an acute dystonic reaction in 1% of adults but 10% of children less than 10 years of age.

a) Antipsychotics b) Tricyclic antidepressants c) Anticonvulsants d) Benzodiazepines e) Opioids

Question 3 The interossei of the hand are supplied by: a) The radial nerve b) The median nerve c) The ulnar nerve

Answer: A When given in high doses in late pregnancy, antipsychotic agents have caused prolonged neurological disturbances in the newborn infant. Anticonvulsants are associated with congenital defects rather than neurological sequelae. Benzodiazepines can cause hypotonia, respiratory depression and hypothermia in the newborn infant if used during labour in high doses. Withdrawal symptoms in neonates have been reported with prolonged maternal use of tricyclic antidepressants. Opioid analgesics may cause respiratory depression in the newborn infant. Withdrawal symptoms in the newborn have been reported with prolonged use of opioids.

d) All of the above e) None of the above

Answer: C The nerve supply of the interossei of the hand is from the deep branch of the ulnar nerve (spinal root C8-T1).

Question 4 Coral is 55 years old. Her periods ceased 8 years ago. She now presents with vaginal itch, discharge and soreness. The MOST LIKELY cause is:

Question 2

a) Candida infection

A 21 year old man walks into your surgery with his head tilted sideways, his eyes rolled up and his tongue sticking out. He speaks with difficulty but says that he has been 'stuck' in this position since taking a new medicine a few hours ago 'for his nerves'. You should administer:

b) Vaginal atrophy c) Vaginal dermatoses d) Gardnerella infection e) None of the above

a) Diazepam b) Benztropine c) Chlorpromazine d) Phenytoin e) Haloperidol

Answer: B 50% of women suffer symptoms of urogenital atrophy in the postmenopausal years. Atrophy of the vaginal mucosa can lead to vaginal dryness, soreness, pruritus and discharge.

Answer: B

Question 5

The presentation described is an acute dystonic reaction, which involves a spasmodic torticollis, (where the head is pulled and held to the left or right by one or other sternomastoid), upward drawn eyes and an open mouth (oromandibular dystonia). It may occur (particularly in young men) within a few days of starting a neuroleptic medication. Treatment is with the anticholinergic medication eg. Benztropine or diphenhydramine. If the patient has a contraindication to anticholinergics then diazepam may be helpful. The dystonias are a group of disorders involving prolonged spasms of muscle contraction. Spasmodic torticollis is one type, as is trismus (clenched jaw) and Blepharospasm (involuntary contraction of the

22 year old Melanie presents with a 2 day history of a vaginal discharge, urinary frequency and dyspareunia. Which organism is the MOST LIKELY cause of this presentation? a) Gardnerella b) HIV c) Chlamydia trachomatis d) Giardia lamblia e) Hepatitis B

Answer: C Chlamydia trachomatis is an intracellular bacterium. Genitourinary symptoms caused by this organism include cervical discharge, cervical bleeding, menstrual change, abdominal pain, fever, nausea, vomiting, urinary frequency and dysuria. Infection rates are highest in the 15-25 year old age group. It is important to remember Chlamydia may be initially asymptomatic especially in women and is the leading cause of preventable infertility and ectopic pregnancy. Although it is important to test for HIV, Melanie's symptoms are not consistent with a seroconversion type illness. Hepatitis B also does not present with gynaecological symptoms. Gardnerella is a simple vaginal infection with no clinical sequelae. Giardia is a single celled organism which infects the small intestine. Question 6 Mr Brown is 70 years old and suffers with severe pain in his back from osteoarthritis. He also suffers with emphysema and has had prednisolone for exacerbations of this disease. He is currently on warfarin for a cardiac arrhythmia and is also taking lansoprazole as a maintenance dose after a duodenal ulcer was diagnosed 4 years ago. Which of the following DOES NOT increase the risk of gastrointestinal side effects from nonsteroidal anti-inflammatory drugs (NSAIDs)? a) Concurrent use of prednisolone b) Previous duodenal ulcer c) Concurrent use of anticoagulant d) The fact that he is over 70 years of age e) Male gender

Answer: E Risk factors for gastro-intestinal side-effects of NSAIDs include age greater than 65 years, previous history of peptic ulcer and co-administration of prednisolone and anti-coagulants. Male gender is not a risk factor.

tightening of the skin with increased risk of the development of squamous cell carcinoma.

Question 8 Which of the following is INCORRECT? a) The incidence Australia is 1:200

of

haemochromatosis

in

b) C282Y homozygotes account for more than 90% of haemochromatosis in Australia c) The majority of patients with one copy each of the C282Y and H63D mutation never develop haemochromatosis d) 90% of C282Y homozygotes develop symptoms of the disorder at some stage in their lives e) Carriers of one copy of the altered gene are generally healthy

Answer: D Studies estimate that up to 50% of C282Y homozygotes will remain symptom free throughout life.

Question 9 You are called to see a 78 year old woman with a threeweek history of headaches and depressive symptoms. She relates that 24 hours ago the vision in her right eye suddenly dimmed. Today the visual acuity in the eye is limited to perception of hand movements only, compared with 6/6 in the left eye. She is not known to be a diabetic and her blood pressure is only minimally elevated. Which of the following possible causes of her visual loss require IMMEDIATE investigation and treatment to prevent blindness in the other eye? a) Detached retina b) Central retinal artery occlusion

Question 7

c) Central vein occlusion

A 55 year old woman presents with an itchy purple papular rash on her wrist. She is not on any medications. This is MOST LIKELY to be:

d) Acute glaucoma e) Temporal arteritis

a) Lichen Planus b) Lichen Simplex Chronicus c) Lichenoid eruption d) Lichen Sclerosus e) Lichenification

Answer: A Lichen Planus is an epidermal inflammatory disorder of unknown cause characterised by pruritic, violaceous, flat tipped papules mainly on the wrists and legs. Lichen Simplex Chronicus is a thickening of the skin as a result of constant scratching, the process of which is called lichenification. Lichen sclerosus is an unexplained vaginal dystrophy characterised by itch and hypertrophy. A lichenoid eruption is most typically due to a drug reaction. Lichen Sclerosis et Atrophicus is a condition of the penis and vulva producing atrophy and

Answer: E Temporal arteritis is an uncommon disease of the elderly and is characterised by the classic complex of fever, anaemia, high ESR and headaches in an elderly person. It is closely associated with polymyalgia rheumatica. Temporal arteritis is the most common manifestation of a systemic vasculitis. Headache is the predominant symptom and may be associated with a thickened or nodular artery. A serious complication, as described in this lady, is ocular involvement - ischaemic optic neuritis. Most patients have head or eye symptoms for months before objective eye involvement. Acute glaucoma causes a red, painful eye, reduced vision and a fixed, mid- dilated pupil which may be slightly ovoid. The pain may be severe and associated with nausea and vomiting. Acute glaucoma may be preceded by blurred vision or haloes around lights. It is a uniocular attack due to blockage of drainage of aqueous fluid from the anterior chamber via the canal of Schlemm. Urgent treatment with hyperosmotic agents is necessary to reduce the intraocular pressure.

Question 10 A 21 year old female patient presents with lower abdominal pain and tenderness at 14 weeks of gestation. Her temperature is 38.5 degrees Celsius. The most important diagnosis to EXCLUDE is: a) Pyelonephritis b) Threatened abortion c) Ectopic pregnancy d) Degeneration of a uterine fibroid e) Appendicitis

Answer: E Appendicitis is the commonest surgical emergency and has its maximum incidence between 20 and 30 years of age. In pregnancy it occurs mainly in the second trimester. Pain is generally higher and more lateral than typical appendicitis. Ectopic pregnancy occurs approximately one in every 100 clinically recognised pregnancies. The classical triad of ectopic pregnancy includes amenorrhoea (65-80%), lower abdominal pain (95+%) and abnormal vaginal bleeding (65-85%). Degeneration of a uterine fibromyoma typically occurs in the second trimester of pregnancy and is due to ischaemic necrosis. In threatened abortion there is vaginal bleeding. Pain is usually not a significant feature unless the cervix is beginning to open. Pyelonephritis can mimic acute appendicitis in pregnancy.

Question 12 An elderly patient with rheumatoid arthritis has been on prednisolone in a dose of 12.5 mg daily for 8 years. After a fall, she complains of severe pain over her lower thoracic spine and is found to be very tender over T11 and T12. A radiograph shows 'cod-fish' vertebrae, with generalised radiolucency of bones and collapse of two vertebral bodies. Examination of her serum is MOST LIKELY to show: a) High calcium, low phosphate, raised alkaline phosphatase b) Low calcium, high alkaline phosphatase

phosphate,

normal

c) Low calcium, low phosphate, raised alkaline phosphatase d) Low calcium, normal phosphate, normal alkaline phosphatase e) Normal calcium, normal phosphate, normal alkaline phosphatise

Answer: E It is most likely the patient has steroid-induced osteoporosis. Plasma calcium, phosphate and alkaline phosphatase are all normal in osteoporosis. If there are multiple recent fractures, serum alkaline phosphatase may be elevated.

Question 13 Question 11 A 42 year old man presents with a recurrent severe hemicranial nocturnal headache which lasts for 60 minutes and occurs regularly every 3 weeks. The headache is accompanied by a blocked nose and watering eye. The MOST LIKELY diagnosis is: a) Chronic paroxysmal hemicrania b) Migraine variant headache

Which of the following may be a good reason for stopping breast feeding: a) Inverted nipples b) Cracked nipples c) Engorged breasts d) Mastitis e) Breast abscess

c) Chronic sinusitis d) Cluster headache

Answer: E

e) Trigeminal neuralgia

Frequent, unrestricted breast feeding day and night is a treatment for breast engorgement. Improved breast drainage is important in mastitis and breastfeeding should not be stopped. Breastfeeding can be continued in the presence of cracked and inverted nipples. Temporary stoppage of breastfeeding on the affected side is necessary while surgical drainage of a breast abscess is carried out.

Answer: D Cluster headache has a four-fold higher incidence in men than women. It is characterised by constant unilateral orbital pain, with onset usually within 2-3 hours of falling asleep. The pain is intense and steady with lacrimation, blocked nostril then rhinorrhoea and sometimes miosis, ptosis, flush and oedema of the cheek, all lasting approximately an hour or two. It tends to occur nightly for several weeks or a few months, followed by complete freedom for months or even years. The response to inhaled oxygen can be dramatic. Chronic paroxysmal hemicrania is similar to cluster headache in presentation except that it is more common in women, attacks occur at any time day or night lasting 2-25 minutes and it always responds to indomethacin. Migraine variants include a range of presentations that are more common in children and often go on to develop a more typical migraine presentation in adulthood.

Question 14 A 40 year old physician has had muscle twitching for one year. The twitching occurs predominantly in the lower extremities. A neurological and general physical examination is negative except for these twitchings. What is the MOST LIKELY diagnosis? a) Benign fasciculations b) Hypoparathyroidism c) Amyotrophic lateral sclerosis d) Paramyoclonus multiplex e) Dystrophia myotonica

Answer: A Diseases of motor neurons or their proximal axons are often associated with fasciculations, the spontaneous firing of an entire motor unit. Fasciculations occur at times in most normal individuals, and unless weakness is present, are seldom of any significance. Myoclonus is a descriptive term for very brief, involuntary, random muscular contractions. Myoclonus may involve a single motor unit and simulate a fasciculation, or it may involve groups of muscles that displace the limb. In Dystrophia Myotonica there is intellectual impairment, amongst other associated deficits, and the weakness starts with the face and extremities. Amyotrophic lateral sclerosis (ALS) is characterised by a progressive loss of motor neurons, both upper and lower motor neurons. The first evidence is an insidiously developing asymmetric weakness. Hypoparathyroidism often presents with neurologic involvement - manifesting as tetany - eg. Chvostek's sign.

intercourse should take place prior to the temperature rise. The purpose of having intercourse on alternate days is to maximise the sperm count on each ejaculation.

Question 17 Sudden onset of unilateral orbital pain, photophobia, lacrimation and blepharospasm suggests a diagnosis of: a) Open-angle glaucoma b) Anterior uveitis c) Temporal arteritis d) Unilateral blepharitis e) Vitreous haemorrhage

Answer: B

Question 15 Which of the following is of LEAST value in assessing the level of active joint inflammation ('disease activity') in rheumatoid arthritis? a) Duration of morning stiffness b) Presence of tiredness, malaise and weight loss c) Degree of elevation of the erythrocyte sedimentation rate (E.S.R.) d) Presence of rheumatoid factor in the blood e) Strength of hand grip

Anterior uveitis (acute iritis) presents with pain of acute onset, photophobia, blurred vision, lacrimation, circumcorneal redness (ciliary congestion) and a small pupil (initially from iris spasm). Talbot's test is positive: pain increases as the eyes converge (and pupils constrict). The slit lamp reveals white precipitates on the back of the cornea and anterior chamber pus (hypopyon). Open angle glaucoma is painless and largely asymptomatic until there is visual field loss; temporal arteritis causes pain in the temporal area, not in the orbit; blepharitis is inflammation of the eyelids; vitreous haemorrhage can present with visual field loss depending on the size of the haemorrhage and it is generally painless.

Question 18

Answer: D Rheumatoid factor (RF) is useful in the diagnosis of rheumatoid arthritis and is of prognostic significance; however fluctuations in RF titre have poor correlation with disease activity. Constitutional symptoms such as tiredness, malaise, weight loss and morning stiffness are good predictors of disease activity.

A patient presents with acute gout but has a low serum uric acid. Which of the following drugs can be responsible for a low serum uric acid? a) Colchicine b) Prednisolone c) Indomethacin

Question 16

d) Naproxyn

In a couple trying to conceive, intercourse should take place:

e) Salicylates

a) As soon temperature

as

there

is

a

rise

in

basal

b) 14 days after the onset of the next period c) On alternate temperature rise d) On alternate temperature rise

days

prior

days

to

after

the the

basal basal

e) Daily after the basal temperature rise

Answer: C Ovulation occurs on day 14 (plus or minus 2 days) before the onset of the next menses. Basal body temperature rises around the time of ovulation and remains elevated for at least 10 days. By the time the temperature rise is detected by the patient, they are probably in the infertile phase of the menstrual cycle, so

Answer: E Up to 30% of patients with acute gout have a normal serum uric acid at presentation. This may be due to precipitation into crystals in the synovium, an effect of a drug the patient has taken for analgesia. Colchicine, steroids and NSAIDS reduce inflammation in affected joints but have no effect on serum uric acid levels. Salicylates eg aspirin at doses less than 2g/day increase serum uric acid levels, but at doses above 4g/day reduce serum uric acid levels.

Question 19 The 'benign' jaundice that occasionally occurs in the third trimester of pregnancy is most CLOSELY related to: a) Decreased circulating amounts of albumin to bind bilirubin

b) A specific defect in glucuronide conjugation

Question 22

c) An autoimmune process

The serum phosphate is MOST LIKELY to be low in which of the following disorders:

d) High oestrogen and progesterone levels e) The use of diuretics in the third trimester of pregnancy

a) Osteoporosis b) Osteomalacia c) Multiple myeloma

Answer: D

d) Polymyalgia rheumatica

In a small number of pregnant women, an intrahepatic cholestasis may appear. It usually occurs in the third trimester of pregnancy and presents as jaundice and pruritus. These patients probably have an increased susceptibility to the hepatic effects of oestrogenic and progestational hormones.

e) Paget's disease

Question 20 Accommodation of accompanied by:

the

eyes

to

NEAR

objects

is

Answer: B Patients with osteomalacia almost always have hypophosphataemia. Serum phosphate is normal in osteoporosis, Paget's disease and polymyalgia rheumatica (PMR). Multiple myeloma may have a normal or slightly raised phosphate level.

a) Loss of binocular vision

Question 23

b) Constriction of the pupils

Which of the following is NOT TRUE regarding asymptomatic bacteriuria in pregnant women?

c) Divergence of the visual axes d) Decreased refractive (dioptric) power e) Relaxation of the ciliary muscles

a) The incidence is 3-7% of all pregnancies b) Defined arbitrarily as greater than 100,000 bacteria/ml c) One of the common Streptococcus Faecalis

Answer: B Normal pupillary responses consist of prompt, symmetric constriction (miosis) on exposure to light or on attempted near convergence. Accommodation involves the active changing of lens shape to focus near objects. Light detection by the retina is passed to the brain via the optic nerve and pupil constriction is mediated by the third cranial nerve (parasympathetic). The sympathetic nervous system is responsible for pupil dilatation via the ciliary nerves.

Question 21 Which finding in this report of a cerebrospinal fluid (C.S.F.) examination is INCONSISTENT with the other results? a) Elevated protein

organisms

is

d) Responds to a short course of oral antibiotics e) Is a clinically unimportant phenomenon

Answer: E Approximately 6% of women have asymptomatic bacteriuria in pregnancy, where there are more than 100,000 bacteria per ml of urine in a mid-stream urine sample. 30% of patients with asymptomatic bacteriuria will subsequently develop acute pyelonephritis. There is an association between pyelonephritis, low birth weight and prematurity. Treatment is with an appropriate course of antibiotics. Recurrence can occur in 35% of cases. The predominant organisms in subclinical or clinical urinary tract infections in pregnancy are E. coli, Strep faecalis, Aerobacter, Klebsiella and Proteus.

b) Normal chloride c) Elevated glucose

Question 24

d) No red cells

With regard to Tietze's syndrome, which of the following statements is INCORRECT? Tietze's syndrome is:

e) Gross leucocytes

excess

of

polymorphonuclear

Answer: C A lumbar puncture consistent with a diagnosis of pyogenic meningitis contains excessive polymorphs, with protein at 1.5g/l (aseptic less than 1.5g/l) and glucose at less than 2/3 the plasma level. There are no red cells unless it is a bloody tap (ie. artefact due to rupture of blood vessel).

a) Painful, tender swelling of one or more costochondral joints b) The cause of up to 30% of chest pain presentations to emergency departments c) Most commonly diagnosed before the age of 40 years d) Sometimes associated with the development of rheumatoid arthritis later in life e) Often associated with repeated minor chest trauma such as harsh coughing

Answer: D

Question 27

The predominant feature of Tietze's syndrome (also called costochondritis) is that of a painful costochondral junction with a palpable swelling. It often occurs in patients with persistent cough and is aggravated by coughing or deep breathing. The 2nd costochondral junction is most commonly affected. Women are more often affected than men. The usual onset is before age 40 years. In the assessment of people presenting with chest pain, 10-30% will have costochondritis, even in childhood. Although many other conditions such as rheumatoid arthritis may present with involvement of the costochondral joints, there is no evidence that Tietze's syndrome will progress to any other illness. It usually follows a fluctuating course with eventual resolution. Management might include analgesics, antiinflammatory drugs, local heat/ice, avoidance of triggers and local steroid injection.

Marion aged 56 years complains of constipation, tiredness, lethargy and lack of energy. She looks puffy in the face, is faintly yellowish in colour and has a pulse of 56 beats/minute regular. Her full blood count shows her haemoglobin is 9.7 g/dl, she has a macrocytosis, lymphocytosis and a raised erythrocyte sedimentation rate (ESR). The MOST LIKELY diagnosis is: a) Pernicious anaemia b) Carcinoma of the bowel c) Alcoholism with malnutrition d) Hypothyroidism e) Lymphocytic leukaemia

Answer: D Question 25 A mother develops a fever of 38.2 degrees Celsius 3 days after the delivery of her baby. Which of the following is the MOST LIKELY cause? a) Endometritis b) Dehydration c) Breast engorgement d) Deep venous thrombosis e) Urinary tract infection (UTI)

Both pernicious anaemia and primary autoimmune hypothyroidism can cause lethargy, a yellow tinge to the skin, lymphocytosis and a macrocytic anaemia with a raised ESR. They often occur simultaneously and are more common in females. However constipation and bradycardia, with puffiness of the face (myxoedema) are more indicative of hypothyroidism, making this the MOST likely diagnosis in this case. One would expect a microcytic anaemia with carcinoma of the bowel due to chronic occult bleeding and more signs of liver disease with alcoholism and malnutrition. Lymphocytic leukemia may cause lethargy, lymphocytosis and jaundice, but none of the other signs.

Answer: A Puerperal infection affects 2-8% of pregnant women and presents with a temperature of greater than 38 degrees Celsius after the first 24 hours post-partum. In up to 75% of cases the cause is genital tract infection, with endometritis most common, especially in the context of a prolonged, complicated labour. Dehydration and breast engorgement may cause a mild pyrexia up to 37.5 degrees. Thrombosis may also cause a low-grade temperature initially although a septic pelvic thrombophlebitis would present later with high fever. UTI occurs in 2-4% of post-partum women and usually presents with low grade fever unless there is pyelonephritis.

Question 26 In which spinal cord segments are the motor neurones responsible for the knee-jerk located? a) L1,L2 b) L2,L3 c) L3,L4 d) L4,L5 e) L5,S1

Answer: C The four muscles of the anterior thigh join to form a common tendon which inserts into the patella. These muscles are supplied by the femoral nerve, spinal roots L3, 4. The quadriceps tendon continues distal to the patella as the patellar tendon and it is this tendon which is stretched in the knee-jerk. Stretching the muscle spindles causes a reflex quadriceps contraction to limit the tendon stretch.

Question 28 In which of the following conditions of the colon is malignant change MOST LIKELY to occur? a) Adenomatous polyp b) Melanosis coli c) Diverticulitis d) Familial polyposis coli e) Ulcerative colitis

Answer: D Familial Intestinal Polyposis occurs in 1/8000 to 1/14000 people in western countries and 50 percent have hundreds to thousands of polyps by age 16. 90% of affected people will develop carcinoma of the colon by age 45. Between 10% and 20% of adenomatous polyps show histological evidence of malignancy when removed. The primary genetic defect occurs in the apoptosis gene p52. The lifetime risk of malignant change in untreated ulcerative colitis is 12%. Melanosis coli is due to laxative abuse and is not pre-malignant in itself. The association between carcinoma of the colon and melanosis coli is probably due more to chronic constipation rather than the staining of the bowel wall from laxatives. Diverticulitis is not associated with malignant change.

Question 29 A 25 year old epileptic woman who had been in good health was admitted to hospital in a comatose condition after the ingestion of 2.0 g of phenobarbitone. On

examination, most reflexes appeared to be intact and there was no depression of respiration. Which of the following forms of therapy would be MOST appropriate? a) Urgent peritoneal dialysis

c) Chlamydia trachomatis d) Neisseria gonorrhoeae e) Streptococcus faecalis

b) Alkalinisation of urine

Answer: C

c) Prophylactic antibiotics

The most common aetiology of epididymitis in males less than 35 years of age is sexually transmitted diseases, and Chlamydia is much more common than the gonococcus. E.coli can be a causative factor in urinary tract infections especially in older men with urinary outflow obstruction (benign prostatic hypertrophy etc). Mumps orchitis and epididymitis are now rare because of widespread vaccination.

d) Administer analeptics e) Give IV corticosteroids

Answer: B Phenobarbitone is a long-acting barbiturate. Peak plasma levels occur within 2-4 hours. It exerts its effects through depression of the central nervous system. It is a weak acid and is 50% protein bound. About 75% of the dose is metabolised and the other 25% excreted unchanged by the kidneys. The half-life in over-dose is 80-120 hours. Initial management of barbiturate overdose requires prompt gastrointestinal decontamination. Barbiturates are well absorbed by activated charcoal. For all barbiturates, attention should be given to hemodynamic and respiratory support, correction of temperature and electrolyte derangement, and monitoring for pulmonary complications. Renal elimination of phenobarbitone is enhanced by alkalinisation of urine to pH 8 and fluid administration with or without mannitol to enhance diuresis.

Question 30 A 32 year old man with ankylosing spondylitis presents to you for management of his disease. Which of the following clinical features is he LEAST LIKELY to complain about? a) Sudden development of dull low back pain b) Attacks of pain and photophobia in one eye c) General malaise, fatigue and weight loss d) Morning stiffness lasting a few hours e) Bilateral hip and shoulder pain

Answer: A The key features of ankylosing spondylitis are the insidious onset of back pain and morning stiffness which improves with exercise, lasting for more than 3 months. Back pain is the most common presenting symptom, and it occurs predominantly in young adults. Approximately 25-35% have an arthritis in the hips and shoulders. Some also have an asymmetric arthritis of other joints. Younger patients often present with a peripheral enthesitis. Older patients may have more constitutional symptoms such as fatigue. Up to 30% will have episodes of acute anterior uveitis and many will have other extra-articular features.

Question 32 Jane is brought into the surgery after being struck in the eye with a tennis ball. On examination you note blood in the anterior chamber of the eye. Which of the following statements regarding her management is INCORRECT? a) Aspirin inflammatories contraindicated

and non-steroidal drugs (NSAIDs)

antiare

b) The main treatment goal is prevention of secondary haemorrhage and glaucoma c) Topical mydriatics should be used to permit examination of the posterior chamber d) Tranexamic acid (Cyklokapron) may be used to stabilise clot formation and prevent rebleeding e) Urgent ophthalmological review is necessary to exclude other ocular damage

Answer: C Management of hyphaema is directed at prevention of secondary haemorrhage within the orbit which carries a high risk of severe glaucoma. Therefore strict bed rest is essential with both eyes covered to reduce eye movements. Aspirin and NSAIDs are contraindicated. Mydriatics would not be used due to the risk of causing the iris to rebleed. An urgent ophthalmological assessment is essential. Ongoing review will check for secondary bleeding (20% occur within 2-3 days), glaucoma and corneal staining. Tranexamic acid may be used to stabilise the clot. Glaucoma may develop months or years after the hyphaema has resolved.

Question 33 The MOST LIKELY venous source of fatal pulmonary embolism is: a) Iliofemoral b) Subclavian c) Saphenous

Question 31

d) Pelvic

In men less than 35 years of age the MOST COMMON causative organism in epididymitis is:

e) Popliteal

a)Mumps virus b) Escherichia coli

Answer: A Most pulmonary emboli arise from proximal deep vein thrombosis (deep veins of lower limb, pelvis and inferior

vena cava). Less frequently, thromboses of the upper arm are the source. Saphenous vein thrombosis seldom results in clinically obvious pulmonary embolism. Also, in order for the thrombus to cause fatality, it has to be large enough to either cause obstruction in the right atrium or of the right ventricular outflow tract. It therefore would have to originate in a large vein.

Question 34 A patient with moderate to severe emphysema would demonstrate all of the following physical signs EXCEPT: a) Distant heart sounds b) A prominent pulmonary second sound c) A narrow intercostal angle d) Prolonged expiration e) Hyperresonance to pulmonary percussion

diabetics will eventually require insulin therapy, the early introduction of a small dose of intermediate-acting insulin before bed will often markedly improve control and ease anxiety about managing injections.

Question 36 In trigeminal neuralgia all of the following statements are true EXCEPT: a) It is most common in middle-aged and elderly persons b) Multiple sclerosis must be considered in younger people c) There is an associated weakness in the muscles of mastication d) The severe, lancinating pain may last for up to two minutes e) It usually carbamazepine

responds

well

to

regular

Answer: C Pulmonary emphysema is one of the obstructive types of pulmonary diseases which leads to progressive hyperinflation of the chest. As a result heart sounds become distant, percussion becomes hyper-resonant and the intercostal angle enlarges as the chest becomes more barrel shaped. Pulmonary hypertension leads to a louder second (P2) heart sound. Prolonged expiration or a prolonged forced expiratory time is an important sign in the obstructive airways group of diseases.

Question 35 Mrs Leung, aged 68 years, has Type 2 diabetes for which she takes metformin 850mg tds and gliclazide 160mg bd. Her blood pressure is well controlled on candesartan 16mg daily. She has no evidence of heart failure and is otherwise well. Her HbA1c is now 9.2% (nondiabetic range
View more...

Comments

Copyright ©2017 KUPDF Inc.
SUPPORT KUPDF